Mechanical Properties of Fluids

You might also like

Download as pdf or txt
Download as pdf or txt
You are on page 1of 33

CHAPTER > 10

Mechanical
Properties of Fluids
KEY NOTES
Å Fluids are those substances which can flow. Liquids and Å Variation of pressure with depth The pressure p at depth
gases falls in the category of fluids. below the surface of a liquid open to the atmosphere is
greater than atmospheric pressure p a by an amount ρgh.
Pressure and Pascal’s Law i.e. Pressure p = p a + ρgh
Å When an object is submerged in a fluid at rest, the fluid Å The excess of pressure, p − p a at depth h is called a gauge
exerts a force on its surface. This force is perpendicular to the pressure.
surface in contact with it. Å The liquid pressure is the same at all points at the same
Å The force exerted by a liquid at rest per unit area of the horizontal level (same depth). The result is appreciated
surface in contact with the liquid is called as pressure. through the example of hydrostatic paradox.
Force ( F)
Pressure ( p) = Å The pressure of the atmosphere (atmospheric pressure) at
Area ( A) any point is equal to the weight of a column of air of unit
It is a scalar quantity and its SI unit is Nm −2 . cross-sectional area extending from that point to the top of
the atmosphere.
Å Density is defined as the ratio of the mass of a body to its
volume.
Å Atmospheric pressure is measured with mercury barometer
m accurately. The mercury column in the barometer has a
ρ= height of about 76 cm at sea level equivalent to one
V
atmosphere (1 atm).
where, ρ = density, V = volume and m = mass.
1 atm = 1.013 × 105 Pa
Å It is a scalar quantity and its SI unit is kg m −3 .
Å Pascal’s law It states that, the change in pressure at one point 1 torr = 133 Pa, 1 bar = 105 Pa
of the enclosed liquid in equilibrium at rest is transmitted Å An open tube manometer is a useful instrument for
equally to all other points of the liquid in all directions. measuring pressure differences.
Å Pressure exerted by a liquid column,
p = ρgh Hydraulic Machines
where, h = height of liquid column,
Å When external pressure is applied on any part of a fluid
contained in a vessel, it is transmitted undiminished and
g = acceleration due to gravity
equally in all directions. This is another form of the Pascal’s
and ρ = density of liquid.
law and it has many applications in daily life.
Å Hydraulic lift and hydraulic brakes are based on the of the liquid remains constant at every cross-section of
Pascal’s law, in which fluids are used for transmitting the tube.’’
pressure. 1
p + ρv 2 + ρgh = constant
Å Hydraulic lift is used to support or lift heavy objects based 2
on the application of Pascal’s law. It is a force multiplying p v2
device with a multiplication factor equal to the ratio of the or + + h = constant
ρg 2g
areas of the two pistons.
p v2
Archimedes’ Principle where, = pressure head, = velocity head
ρg 2g
Å When a body is immersed partially or fully in a liquid, then and h = gravitational head.
resultant upward force on the body is called buoyant force.
Å Bernoulli’s equation ideally applies to fluids with zero
Å According to Archimedes’ principle, ‘‘when a body is viscosity or non-viscous fluids.
partially or fully immersed in a fluid at rest, the fluid exerts
an upward force of buoyancy which is equal to the weight Speed of Efflux : Torricelli’s Law
of the displaced fluid. Å The outflow of a fluid is called efflux and the speed of
Å Due to this upward force, the weight of the body appear to the fluid coming out is called speed of efflux.
be decreased. Å When tank as shown below is closed, the speed of efflux
Å If total volume of object is Vs and a part Vp of it is submerged 2( p − p a)
in the fluid, then is given by v1 = 2gh +
ρ
weight of displaced fluid = weight of object
where, ρ = density of liquid.
ρ Vp
ρ s gVs = ρ f gVp ⇒ s =
ρ f Vs
A2
2 p 2 =p
Flow of Liquids
Å If the velocity of fluid particles at any time does not vary h
with time, the flow is said to be steady or streamline flow. A1
1
Å The path followed by a fluid particle in streamline flow is y2 v1
known as streamlines. pa
Å Velocity of particles in streamline is along the tangent to the y1
curve at that point.
Å The flow of fluid in which velocity of all particles crossing a
given point is not same and the motion of the fluid is
irregular is called turbulent flow. Special case
Å If the liquid flows over a horizontal surface in the form of When the tank is open to the atmosphere, then
layers of different velocities, then the flow of liquid is called
laminar flow. p = pa
Å Equation of continuity It states that ‘‘when an ∴ v1 = 2gh
incompressible and non-viscous fluid flows steadily This is also the speed of a freely falling body and this
through a tube of non-uniform cross-section, then the equation represents Torricelli’s law.
product of area of cross-section and velocity of flow is same Å The horizontal distance covered by the liquid coming out
at every point in the tube, i.e. A 1v1 = A 2v 2 of the hole is called range and is given by
where, A = area of cross-section and v = velocity of flow.
R = 2 h( y 2 − y1 )
Bernoulli’s Principle Venturi-meter
Å According to this principle, ‘if an ideal fluid is flowing in
It is a device which is used to measure the flow speed of
streamlined flow, then total energy, i.e. sum of pressure
incompressible fluid. According to given figure, speed of
energy, kinetic energy and potential energy per unit volume
fluid at wide neck is given as

KEY NOTES
A
a Viscosity
Å The property of a fluid by virtue of which an internal
frictional force acts between its different layers, which
2 opposes their relative motion is called viscosity.
1 Å The velocities of layers increase uniformly from bottom
h
(zero velocity) to the top layer (velocity v).
Å The coefficient of viscosity for a fluid is defined as the
ratio of shearing stress to the strain rate.
F/ A Fl
η= =
v /l vA
−1 / 2
2 ρ m gh  A  
2
Its SI unit is poiseuille (PI).
v1 = ⋅   − 1
ρ  
 a  The viscosity of liquids decreases with temperature, while
Å

it increases in the case of gases.


where, ρ m = density of liquid contained in U-tube,
Å Stokes’ Law There is a viscous drag force F on a sphere of
ρ = density of fluid radius r moving with velocity v through a fluid of
and h = difference in height in U-tube. viscosity η. It can be expressed as
Blood Flow and Heart Attack F = 6πηrv
Bernoulli’s principle helps in explaining blood flow in artery. Å Terminal Velocity The maximum constant velocity
The artery may get constricted due to the accumulation of acquired by the body while falling through a viscous fluid
plaque on its inner surface. Due to this, the flow of blood is called terminal velocity.
increases in this region, resulting in decrease in pressure. 2 r 2 (ρ − σ) g
v= ×
The decreasing pressure makes the artery to collapse, 9 η
resulting in heart attack. where, r = radius of the spherical body,
Dynamic Lift v = terminal velocity,
Å It is the force that acts on a body, such as airplane wing, a η = coefficient of viscosity of fluid,
hydrofoil or a spinning ball by virtue of its motion through ρ = density of the spherical body
a fluid. There arises following two cases, which can be and σ = density of fluid.
explained on the basis of Bernoulli’s principle.
(i) When ball is moving without spin in air, then speed Surface Tension
of air above and below to the ball is streamline, hence Å It is the property of liquid at rest by virtue of which a
pressure difference above and below the ball is zero. liquid surface tends to occupy a minimum surface area
The air, therefore exerts no upward or downward and behaves like a stretched membrane.
force on the ball. Å The surface energy may be defined as the amount of work
(ii) When ball is moving with spin in air, then speed of done in increasing the area of the surface film through
air above and below to the ball is not streamline, unity. It is expressed as
hence pressure difference above and below the ball is Work done in increasing surface area
not zero. Due to difference in velocities of fluid (air) Surface energy =
Increase in surface area
exerts, a net upward force on the ball.
Å Surface tension and Surface energy Surface tension is
Å Magnus Effect When a ball is moving in air with spin, force per unit length (or surface energy per unit area)
then due to difference in the velocities of air results in the acting in the plane of the interface between the plane of
pressure difference between the lower and upper faces the liquid and any other substance. It is also the extra
and there is net upward force on the ball. energy that the molecules at the interface have as
This dynamic lift due to spinning is called Magnus effect. compared to molecules in the interior.
Å Aerofoil or Lift on Aircraft Wing An aerofoil is solid Å The value of surface tension depends on temperatue.
piece shaped to provide an upward dynamic lift when it Å Like viscosity, the surface tension of a liquid usually falls
moves horizontally through air. The cross-section of the with temperature.
wings of an aeroplane looks like the aerofoil.

KEY NOTES
Angle of Contact Capillary Rise
Å The angle subtended between the tangent drawn at liquid’s Å The phenomenon of rising or falling of liquid in a capillary
surface and tangent drawn at solid surface inside the liquid tube is called capillarity.
at the point of contact is called angle of contact. Å The height of liquid column in a capillary tube is given by
Å At the line of contact, the surface forces between three media 2S cosθ
as shown in Figs. (i) and (ii) must be in equilibrium, h=
rρg
if
where, r is the radius of the capillary tube, θ is the angle of
Sla cos θ + Ssl = Ssa contact and ρ is density of liquid.
where, In capillary, there arises following cases
Sla = surface force of liquid-air interface, (i) When the angle of contact between the liquid and glass
Ssl = surface force of solid-liquid interface is acute, then surface of liquid in the capillary is
and SSa = surface force of solid-air interface. concave. The pressure of the liquid inside the tube, just
Sla at the meniscus (air-liquid interface) is less than the
Sla
atmospheric pressure.
θ (ii) When the angle of contact between the liquid and glass
θ is obtuse, then surface of liquid in the capillary is
Ssa Ssl Ssa Ssl convex. The pressure of liquid inside the tube, just at
(i) (ii) the meniscus (air-liquid interface) is greater than the
(i) If Ssl > Sla, i.e. angle of contact is an obtuse angle for atmospheric pressure.
solid-liquid interface, then liquid does not wet the solid. (iii) When the angle of contact between the liquid and glass
(ii) If Ssl < Sla , i.e. angle of contact is an acute angle for is right angle, the surface of liquid in the capillary tube
solid-liquid interface, then liquid wet the solid. is plane.
(iii) If Sla = Ssa , i.e. angle of contact is right angle for The pressure of liquid inside the tube, just at the
solid-liquid interface. meniscus (air-liquid interface) is equal to the
atmospheric pressure.
Drops and Bubbles Å Detergents and Surface Tension By addition of detergents
One consequence of surface tension is that, the pressure inside in water, surface tension decreases.
p i a spherical drop is more than the pressure outside p o. Å It is favourable to form interfaces like globs of dirt
2S surrounded by detergents and then by water. This kind of
Excess pressure inside a liquid drop, ( p i − p o) =
R process using surface active detergents or surfactants is
4S important not only for cleaning, but also in recovering oil,
Excess pressure inside a soap bubble, ( p i − p o) =
R mineral ores, etc.

Mastering NCERT
MULTIPLE CHOICE QUESTIONS

TOPIC 1 ~ Pressure and Pascal's Law


1 The key property of fluids is that (c) it can easily penetrate the surface
(a) they offer very little resistance to shear stress (d) All of the above
(b) their shape changes 3 When an object is submerged in a fluid at rest, then
(c) they offer very large resistance to shear stress fluid exerts a force on its surface. This force is
(d) Both (a) and (b)
always
2 Metal nails and metal pins are made to have pointed (a) normal to the objects surface
ends because (b) parallel to the objects surface
(a) it transmit large pressure (c) along 45° to the objects surface
(b) it transmit large force (d) None of the above
4 The two thin bones (femurs), each of cross-sectional 10 A uniformly tappering vessel is filled with a liquid of
area 10 cm 2 support the upper part of human body of density 900 kgm −3 . The force that acts on the base of
mass 40 kg. Estimate the average pressure sustained the vessel due to the liquid is (take, g =10 ms −2 )
by the femurs.
(a) 2 × 105 Nm−2 (b) 3 × 104 Nm−2
(c) 2.5 × 103 Nm−2 (d) 6 × 104 Nm−2
0.4 m
5 If two forces in the ratio 1 : 7 act on two pistons of
areas in the ratio 3 : 2, then the pressure exerted by
the forces is in ratio
(a) 2 : 21 (b) 3 : 14 (c) 6 : 7 (d) 4 : 21 Area =2 × 10–3 m2
(a) 3.6 N (b) 7.2 N (c) 9.0 N (d) 14.4 N
6 If two liquids of same masses but densities ρ1 and ρ 2
respectively are mixed, then density of mixture is 11 The heart of a man pumps 5 L of blood through
given by the arteries per minute at a pressure of 150 mm of
ρ1 + ρ2 mercury. If the density of mercury be
(a) ρ = 13.6 × 10 3 kg m −3 and g = 10 ms −2 , then the power of
2
ρ1 + ρ2 heart in watt is CBSE AIPMT 2015
(b) ρ =
2 ρ1 ρ2 (a) 1.70 (b) 2.35
2 ρ1ρ2 (c) 3.0 (d) 1.50
(c) ρ =
ρ1 + ρ2 12 The approximate depth of an ocean is 2700 m. The
ρρ compressibility of water is 45.4 × 10 −11 Pa −1 and
(d) ρ = 1 2
ρ1 + ρ2 density of water is 10 3 kg m −3 . What fractional
compression of water will be obtained at the bottom
7 Pressure at a point inside a liquid does not depend on
of the ocean? CBSE AIPMT 2015
(a) the depth of the point below the surface of the liquid
(b) the nature of the liquid (a) 0.8 × 10−2 (b) 1.0 × 10−2 (c) 1.2 × 10−2 (d) 1.4 × 10−2
(c) the acceleration due to gravity at that point 13 A long cylindrical vessel is half-filled with a liquid.
(d) total weight of fluid in the beaker When the vessel is rotated about its own vertical axis,
8 Pascal’s law states that pressure in a fluid at rest is the the liquid rises up near the wall. If the radius of vessel
same at all points, if is 5 cm and its rotational speed is 2 rotations per
(a) they are at the same height second, then the difference in the heights between the
(b) they are along same plane centre and the sides (in cm) will be JEE Main 2019
(c) they are along same line (a) 0.1 (b) 1.2
(d) Both (a) and (b) (c) 0.4 (d) 2.0
9 Figure given below shows an element in the interior of 14 In a U-tube as shown in a figure, water and oil are in
a fluid at rest. This elemental volume is in the shape of the left side and right side of the tube, respectively.
a right angled prism. Let the elemental volume is small The heights from the bottom for water and oil
enough that we can ignore the effect of gravity, but it is columns are 15 cm and 20 cm, respectively. The
drawn in an enlarged scale for the sake of clarity. density of the oil is [take ρ water =1000 kgm −3 ]
NEET (Odisha) 2019
D
A
θ Fb
Fc
E
θ F Water 20 cm
B C 15 cm Oil
Fa

For equilibrium of elemental volume, which of these


are correct?
(a) 1200 kgm −3 (b) 750 kgm −3
(a) Fb sin θ = Fc (b) Fc cosθ = Fb (c) 1000 kgm −3 (d) 1333 kgm −3
(c) Fb cosθ = Fc (d) Fa cosθ = Fb
15 A U-tube with both ends open to the atmosphere is 19 A hydraulic lift has 2 limbs of areas A and 2A. Force
partially filled with water. Oil which is immiscible F is applied over limb of area A to lift a heavy car. If
with water, is poured into one side until it stands at a distance moved by piston P1 is x, then distance
distance of 10 mm above the water level on the other moved by piston P2 is
side. Meanwhile the water rises by 65 mm from its F
original level (see diagram), the density of the oil is 2A Piston P2
NEET 2017 Piston P1 A
Pa Pa
F
A
10 mm
E Final water level x
65 mm (a) x (b) 2 x (c) (d) 4x
Oil D Initial water level
2
65 mm 20 Two syringes of different cross-sections (without
B C needles) filled with water are connected with a tightly
Water fitted rubber tube filled with water. Diameters of the
smaller piston and larger piston are 1.0 cm and
3.0 cm, respectively. If the smaller piston is pushed in
(a) 650 kg m−3 (b) 425 kg m−3
through 6.0 cm, how much does the larger piston
(c) 800 k g m−3 (d) 928 kg m−3 move out?
16 Find density of ethyl alcohol. Using information (a) 0.67 cm (b) 0.5 cm (c) 0.75 cm (d) 1.00 cm
given in the diagram below JIPMER 2018 21 A car of mass 500 kg is lifted by a hydraulic jack that
consist of two pistons. If the diameter of large and small
pistons are 2 m and 20 cm respectively, then force required
12 cm to lift the car by smaller piston is (take, g = 10 m/s 2 )
10 cm Ethyl alcohol
Water (a) 5000 N (b) 25 N (c) 500 N (d) 50 N
22 Consider a solid sphere of radius R and mass density
 r2 
ρ( r ) = ρ 0 1 − 2  , 0 < r ≤ R. The minimum density of
Hg  R 
a liquid in which it will float is JEE Main 2020
(a) 0.83 gcm −3 (b) 0.5 gcm −3 (c) 1.83 gcm −3 (d) 0.12 gcm −3 ρ0 2 ρ0 2 ρ0 ρ0
(a) (b) (c) (d)
17 A liquid of density ρ is coming out of a hose pipe of 3 5 3 5
radius a with horizontal speed v and hits a mesh. 50% 23 An ice cube floats on water in a beaker with (9/10)th
of the liquid passes through the mesh unaffected 25% of its volume submerged under water. What fraction
losses all of its momentum and, 25% comes back with of its volume will be submerged, if the beaker of
the same speed. The resultant pressure on the mesh water is taken to the moon where the gravity is (1/6)th
will be JEE Main 2019 that on the earth?
1 2 1 2 3 2 9 27 2
(a) ρv2 (b) ρv (c) ρv (d) ρv (a)
10
(b)
50
(c)
3
(d) Zero
2 4 4
24 A cubical block of steel of each side equal to l is
18 A student measures pressure of a gas in a container
floating on mercury in a vessel. The densities of steel
using a mercury manometer and she also measures
and mercury are ρ s and ρ m . The height of the block
atmospheric pressure using a mercury barometer. She above the mercury level is given by
gave following representation  ρ   ρ   ρ   ρ 
p2 (a) l 1 + s  (b) l 1 − s  (c) l 1 + m  (d) l 1 − m 
 ρm   ρm   ρs   ρs 
h2 h3
p1 25 Two non-mixing liquids of densities ρ and nρ(n > 1)
h1 are put in a container. The height of each liquid is h.
p0 A solid cylinder of length L and density d is put in
this container. The cylinder floats with its axis
If p1 = atmospheric pressure,
vertical and length pL (p < 1) in the denser liquid.
and p2 = absolute pressure, then The density d is equal to NEET 2016
(a) gauge pressure = h1 + h2 (b) gauge pressure = h3 − h1 (a) {2 + ( n + 1) p}ρ (b) {2 + ( n − 1) p}ρ
(c) gauge pressure = h3 (d) absolute pressure = h1 (c) {1 + ( n − 1) p}ρ (d) {1 + ( n + 1) p}ρ
Master The NCERT > PHYSICS (Vol-I )

TOPIC 2 ~ Flow of Liquids and Bernoulli's Principle


26 In a streamline flow, 32 An incompressible liquid flows through a horizontal
(a) the speed of a particle always change tube as shown (areas of tubes is marked), then the
(b) the velocity of each particle always remains same at a velocity v of the fluid is
given point
(c) the kinetic energies of all the particles arriving at a v2 = 1.5 ms–1
given point are the same A
(d) the potential energies of all the particles arriving at a
given point are the same v1=3 ms–1 A
27 In a turbulent flow, the velocity of the liquid 1.5
molecules in contact with the walls of the tube is A
(a) zero (b) maximum v
(c) equal to critical velocity (d) may have any value
(a) 3.0 ms −1 (b) 1.5 ms −1 (c) 1.0 ms −1 (d) 2.25 ms −1
28 In a laminar flow, the velocity of the liquid in contact
with the walls of the tube is 33 The cylindrical tube of a spray pump has radius R,
(a) zero one end of which has n fine holes, each of radius r. If
(b) maximum the speed of the liquid in the tube is v, the speed of
(c) in between zero and maximum the ejection of the liquid through the holes is
(d) equal to critical velocity CBSE AIPMT 2015
vR 2 vR 2 vR 2 v2 R
29 In case of streamline flow of a fluid (which is (a) (b) (c) (d)
incompressible), consider these streamlines A, B and n 2 r2 nr2 n 3 r2 nr
C (as shown in the figure) 34 According to Bernoulli’s equation,
p 1 v2
+h+ = constant
P ρg 2 g
Q p 1 v2
The terms, , h and are generally called
A ρg 2 g
B
R
C respectively :
Let, P , Q and R are 3 planes perpendicular to the (a) Gravitational head, pressure head and velocity head
direction of flow of streamlines, then which of the (b) Gravity, gravitational head and velocity head
following is correct? (c) Pressure head, gravitational head and velocity head
(a) mP > mQ > mR , where m = mass flow per second. (d) Gravity, pressure and velocity head
(b) vP > vQ > vR 35 Air is streaming past a horizontal airplane’s wing
(c) nP > nQ > nR , where n = number of fluid particles such that its speed is 120 ms −1 over the upper surface
crossing area per unit time.
and 90 ms −1 at the lower surface. If the density of air
(d) mP < mQ < mR
is 1.3 kgm −3 and the wing is 10 m long and has an
30 An ideal fluid flows (laminar flow) through a pipe of average width of 2 m, then the difference of the
non-uniform diameter. The maximum and minimum pressure on the two sides of the wing is
diameters of the pipes are 6.4 cm and 4.8 cm, (a) 4095.0 Pa (b) 409.50 Pa
respectively. The ratio of the minimum and the (c) 40.950 Pa (d) 4.0950 Pa
maximum velocities of fluid in this pipe is
36 A wind with speed 40 ms −1 blows parallel to the roof
JEE Main 2020
of a house. The area of the roof is 250 m 2 . Assuming
9 81 3 3
(a) (b) (c) (d) that the pressure inside the house is atmospheric
16 256 2 4 pressure, the force exerted by the wind on the roof
31 A liquid flows through a pipe of varying diameter. and the direction of the force will be
The velocity of the liquid is 2 ms −1 at a point, where (ρ air = 1.2 kgm −3 ) CBSE AIPMT 2015
the diameter is 6 cm. The velocity of the liquid at a
(a) 2.4 × 105 N, downwards (b) 4.8 × 105 downwards
point, where the diameter is 3 cm will be
(a) 1 ms −1 (b) 4 ms −1 (c) 8 ms −1 (d) 16 ms −1 (c) 4.8 × 105 N, upwards (d) 2.4 × 105 N, upwards
37 Determine the pressure difference in tube of 41 A small hole of area of cross-section 2 mm 2 is present
non-uniform cross sectional area as shown in figure. near the bottom of a fully filled open tank of height 2 m.
∆p = ?, d1 = 5 cm, v1 = 4 m/s, d 2 = 2 cm. Taking g = 10 m/s 2 , the rate of flow of water through
AIIMS 2019 the open hole would be nearly NEET (National) 2019
(a) 8.9 × 10−6 m 3 /s (b) 2. 23 × 10−6 m 3 /s
d1 (c) 6.4 × 10−6 m 3 /s (d) 12.6 × 10−6 m 3 /s
v1 d2 v2
42 A cylinder of height 20 m is completely filled with
water. The velocity of efflux of water (in ms −1 )
through a small hole on the side wall of the cylinder
∆p near its bottom is
(a) 10 (b) 20 (c) 25.5 (d) 5
(a) 304200 Pa (b) 304500 Pa
(c) 302500 Pa (d) 303500 Pa 43 A hole is made at the bottom of the tank filled with
water (density 1000 kgm −3 ). If the total pressure at
38 A sealed tank has 2-openings as shown below. One at the bottom of the tank is 3 atm (1 atm =10 5 Nm −2 ),
near top and other at near bottom. Let height of water then the velocity of efflux is
filled above the bottom opening is h and a compressor (a) 200 ms −1 (b) 400 ms −1
producing a pressure p is connected to top opening.
(c) 500 ms −1 (d) 800 ms −1
Velocity of water obtained from lower opening is
(take, atmospheric pressure pa such that 44 The applications of venturimeter is/are
p – pa = ρgh) (a) carburetor of an automobile
(b) sprayers
To compressor
(c) filter pumps
p (d) All of the above
45 The flow of blood in a large artery of an anesthetised
pa
dog is diverted through a venturimeter. The wider
part of the meter has a cross-sectional area equal to
v
that of the artery, A = 8 mm 2 .
The narrower part has an area, a = 4 mm 2 and density
of blood, ρ =1. 06 × 10 3 kgm −3 .
(a) 2gh (b) gh
The pressure drop in the artery is 24 Pa, then what is
(c) 2 gh (d) 0
the speed of the blood in the artery?
39 Water stands at a depth H in a tank whose side walls (a) 0.5 ms −1 (b) 0.125 ms −1
are vertical. A hole is made in one of the walls at a (c) 1.25 ms −1 (d) 2.5 ms −1
height h below the water surface. The stream of water 46 Just before “Heart attack”, velocity of blood flow
emerging from the hole strikes the floor at a distance through the affected “artery”
R from the tank, where R is given by (a) increases (b) decreases
(a) R = h ( H − h ) (b) R = h ( H + h ) (c) remains same (d) stopped
(c) R = 2 h ( H − h ) (d) R = 2 h ( H + h ) 47 A ball is moving without spinning in a straight line
through a fluid (as shown)
40 If a small orifice is made at a height of 0.25 m from the
ground as shown in figure below, the horizontal range pA
of water stream will be AIIMS 2019 A

B
1m pB

0.25 m
If p A and pB are pressure values at A and B, then
(a) 46.5 cm (b) 56.6 cm (a) p A < pB (b) pB < p A
(c) 76.6 cm (d) 86.6 cm (c) p A × pB = 1 (d) p A / pB = 1
48 A ball is moving in a straight line through a fluid 49 A fully loaded boeing aircraft has a mass of
which is spinning around its own centre of mass as 3.3 × 10 5 kg. Its total wing area is 500 m 2 . It is in
shown. level flight with a speed of 960 kmh −1 . (i) Estimate
the pressure difference between the lower and upper
surfaces of the wings (ii) Estimate the fractional
increase in the speed of the air on the upper surface of
the wing relative to the lower surface. The density of
air is ρ = 1.2 kgm −3 .
Then, the ball experiences (a) 6.5 × 10+3 Nm−2 , 0.01 (b) 6.5 × 103 Nm−2 , 0.09
(a) an upward force (b) a downward force (c) 6.5 × 103 Nm−2 , 0.08 (d) 2.5 × 103 Nm−2 , 0.02
(c) a leftward force (d) no force at all

TOPIC 3 ~ Viscosity
50 As the temperature of water increases, its viscosity (a) 4 × 10−2 Pa-s (b) 3.45 × 10−3 Pa-s
(a) remains unchanged (c) 5 × 10−2 Pa-s (d) 7 × 10−7 Pa-s
(b) decreases
(c) increases 54 A rain drop of radius 0.3 mm has a terminal velocity
(d) increases or decreases depending on the external pressure in air 1 ms −1 . The viscosity of air is 18 × 10 −5 poise.
Find the viscous force on the rain drops. JIPMER 2018
51 The coefficient of viscosity for hot air is
(a) 5.02 × 10−7 N (b) 1.018 × 10−7 N
(a) greater than the coefficient of viscosity for cold air
−7
(b) smaller than the coefficient of viscosity for cold air (c) 1.05 × 10 N (d) 2.058 × 10−7 N
(c) same as the coefficient of viscosity for cold air 55 Which one shows the variation of the velocity v with
(d) increases or decreases depending on the external pressure time t for a small sized spherical body falling in a
52 We have three beakers A, B and C containing three column of a viscous liquid?
different liquids. They are stirred vigorously and v v
placed on a table, then liquid which is
(a) most viscous comes to rest at the earliest (a) (b)
(b) most viscous comes to rest at the last t t
(c) most viscous slows down earliest but comes to rest at O O
the last v v
(d) All of them come to rest at the same time
(c) (d)
53 A metal block of area 0.10 m 2 is connected to a
0.010 kg mass via a string that passes over an ideal t t
O O
pulley (considered massless and frictionless), as in
figure. A liquid with a film thickness of 0.30 mm is
placed between the block and the table. When 56 A small drop of water falls from rest through a large
released the block moves to the right with a constant height h in air, the final velocity is proportional to
speed of 0.085 ms −1 , find the coefficient of viscosity (a) h (b) h (c) 1/ h (d) h 0
of the liquid.
57 The terminal velocity of a copper ball of radius
Film 2.0 mm falling through a tank of oil at 20°C is
6.5 cms −1 . Compute the viscosity of the oil at 20°C.
Density of oil is 1.5 × 10 3 kgm −3 and density of
copper is 8.9 × 10 3 kg m −3 .
(a) 1 × 10−1 kg ms −1 (b) 9.9 × 10−1 kg ms −1
−2 −1
(c) 24.3 × 10 kg ms (d) 2 × 10−2 kg ms −1
58 If ratio of terminal velocity of two drops falling in air 60 Two small spherical metal balls, having equal masses
is 3 : 4, then what is the ratio of their surface area? are made from materials of densities ρ1 and
JIPMER 2018 ρ 2 ( ρ1 = 8ρ 2 ) and have radii of 1 mm and 2 mm,
2 3 respectively. They are made to fall vertically (from
(a) (b)
3 4 rest) in viscous medium whose coefficient of
(c)
4
(d)
3 viscosity equals η and whose density is 0.1 ρ 2 . The
3 2 ratio of their terminal velocities would be
59 Consider two solid spheres P and Q each of density NEET (Odisha) 2019
−3 79 19 39 79
8 g cm and diameters 1 cm and 0.5 cm, respectively. (a) (b) (c) (d)
72 36 72 36
Sphere P is dropped into a liquid of density
61 A small sphere of radius r falls from rest in a viscous
0.8 g cm −3 and viscosity η =3 poiseuille. Sphere Q is
liquid. As a result, heat is produced due to viscous
dropped into a liquid of density 1.6 g cm −3 and force. The rate of production of heat when the sphere
viscosity η = 2 poiseiulle. The ratio of the terminal attains its terminal velocity, is proportional to
velocities of P and Q is NEET 2018
(a) 3 : 1 (b) 9 : 1 (a) r5 (b) r2
(c) 2 : 4 (d) 4 : 2 (c) r3 (d) r4

TOPIC 4 ~ Surface Tension


62 Surface tension is due to 66 A 10 cm long wire is placed horizontally on the
(a) frictional forces between molecules surface of water and is gently pulled up with a force
(b) cohesive forces between molecules of 2 × 10 −2 N. To keep the wire in equilibrium, the
(c) adhesive forces between molecules surface tension in Nm −1 of water is
(d) Both (b) and (c) (a) 0.1 (b) 0.2
63 The surface tension of a liquid at its boiling (c) 0.001 (d) 0.002
point 67 A wooden stick 2 m long is floating on the surface of
(a) becomes zero water. The surface tension of water is 0.07 Nm −1 . By
(b) becomes infinity putting soap solution on one side of the stick, the
(c) is equal to the value at room temperature surface tension is reduced to 0.06 Nm −1 . The net
(d) is half to the value at room temperature force on the stick will be
64 A liquid film is formed over a frame ABCD as shown (a) 0.07 N (b) 0.06 N
in figure. Wire CD can slide without friction. (c) 0.01 N (d) 0.02 N
Maximum value of mass that can be hanged from CD 68 A thin liquid film is formed between a
without breaking the liquid film is U-shaped wire and a light slider,
A B supporting a weight of 1.5 × 10 −2 N as Film
Liquid film shown in the figure. The length of the
(Surface tension T ) slider is 30 cm and its weight is
D C
negligible. The surface tension of the w
l liquid film is JEE Main 2014
Tl 2Tl (a) 0.0125 Nm−1 (b) 0.1 Nm−1
(a) (b)
g g (c) 0.05 Nm−1 (d) 0.025 Nm−1
g
(c) (d) T × l 69 The shape of drops and bubbles are spherical due to
2Tl
its
65 The force required to separate two glass plates of (a) surface with minimum energy
10 −2 m 2 with a film of water 0.05 mm thick between (b) surface with maximum energy
them, is (surface tension of water is 70 × 10 −3 Nm −1 ) (c) high pressure
(a) 28 N (b) 14 N (c) 50 N (d) 38 N (d) low pressure
70 A spherical drop of radius r is in equilibrium. 76 In a soap bubble, pressure difference is
The extra surface energy, if radius of bubble is 2 S la 4 S la S la 8 S la
(a) (b) (c) (d)
increased by ∆r, is (S = surface tension) r r r r
(a) 4πr ∆r S (b) 8πr ∆r S 77 The excess pressure inside an air bubble of radius r
(c) 2πr ∆r S (d) 10πr ∆r S just below the surface of water is p1 . The excess
71 In figure below, pressure inside a spherical drop is pressure inside a drop of the same radius just outside
more than pressure outside. the surface is p2 . If T is the surface tension, then
(a) p1 = 2 p2 (b) p1 = p2
If a liquid drop is in equilibrium, then the pressure
(c) p2 = 2 p1 (d) p2 = 0, p1 ≠ 0
difference between the inside and outside of the drop
is 78 The surface tension of water at temperature of the
po experiment is 7.30 × 10 −2 Nm −1 . (1 atm pressure
= 1.01 × 10 5 Pa, density of water = 1000 kgm −3 and
pi . ms −2 ). Calculate the pressure inside a bubble
g = 980
r
of radius 1mm.
(a) 3 × 102 Pa (b) 8 × 104 Pa
2S la S la 4S la 2r (c) 1.01 × 105 Pa (d) 7 × 103 Pa
(a) (b) (c) (d)
r r r S la
79 A soap bubble, blown by a mechanical pump at the
72 A small spherical droplet of density d is floating mouth of a tube, increases in volume, with time, at a
exactly half immersed in a liquid of density ρ and constant rate. The graph that correctly depicts the
surface tension T. The radius of the droplet is (take note time dependence of pressure inside the bubble is
that the surface tension applies an upward force on the given by JEE Main 2019
droplet) JEE Main 2020
3T T p
(a) r = (b) r = p
( 2d − ρ )g ( d − ρ )g (a) (b)
T 2T
(c) r = (d) r =
( d + ρ )g 3( d + ρ )g 1 1
t3 t
73 A soap bubble of radius r is blown up to form a
bubble of radius 3r under isothermal conditions. If σ p p
is the surface tension of soap solution, the energy (c) (d)
spent in blowing is
(a) 3 πσr2 (b) 6πσr2 (c) 12πσr2 (d) 64 πσr2 t log (t)
74 Two small drops of mercury, each of radius r, 80 A soap bubble having radius of 1 mm is blown from a
coalesce to form a single large drop of radius R. The detergent solution having a surface tension of
ratio of the total surface energies before and after the . × 10 −2 N/m. The pressure inside the bubble equals
25
change is at a point Z 0 below the free surface of water in a
(a) 1 : 21 / 3 (b) 21 / 3 : 1
container. Taking, g =10 m/s 2 and density of water
(c) 2 : 1 (d) 1 : 2
=10 3 kg/m 3 , the value of Z 0 is NEET (National) 2019
75 A certain number of spherical drops of a liquid of
(a) 10 cm (b) 1 cm (c) 0.5 cm (d) 100 cm
radius r coalesce to form a single drop of radius R and
volume V. If T is the surface tension of the liquid, 81 If the air bubble of radius r is formed at a depth h inside
then CBSE AIPMT 2014 the container of soap solution of density ρ, the total
 1 1 pressure inside the bubble is (here, po denotes the
(a) energy = 3 VT  −  is released atmospheric pressure and σ denotes surface tension)
 r R
JEE Main 2013
(b) energy is neither released nor absorbed
2σ 2σ
 1 1 (a) + hρg (b) − hρg
(c) energy = 4VT  −  is released r r
 r R
2σ 2σ
 1 1 (c) + po + hρg (d) + po − hρg
(d) energy = 3VT  +  is absorbed r r
 r R
82 The lower end of a capillary tube of diameter Which of the following shows the relative nature of
2.00 mm is dipped 8.00 cm below the surface of water the liquid columns in the two tubes?
in a beaker. What is the pressure required in the tube A B A B
in order to blow a hemispherical bubble at its end in
water? (a) (b)
(a) 2 × 105 Pa
(b) 1.01784 × 105 Pa
(c) 3 × 103 Pa A B A B

(d) 2.438 × 105 Pa (c) (d)


83 Water rises to a height h in capillary tube. If the
length of capillary tube above the surface of water is
less than h, then CBSE AIPMT 2015
86 Two capillaries made of same material but of
(a) water rises upto the tip of capillary tube and then starts
overflowing like a fountain different radii are dipped in a liquid. The rise of liquid
(b) water rises upto the top of capillary tube and stays there in one capillary is 2.2 cm and that in the other is
without overflowing 6.6 cm. The ratio of their radii is
(c) water rises upto a point a little below the top and stays (a) 9 : 1 (b) 1 : 9 (c) 3 : 1 (d) 1 : 3
there 87 If M is the mass of water that rises in a capillary tube
(d) water does not rise at all of radius r, then mass of water which will rise in a
84 Find the height of liquid in capillary tube, if surface capillary tube of radius 2r is JEE Main 2019
tension of liquid = S, radius of capillary tube = r and (a) 2M (b) 4M (c)
M
(d) M
acceleration due to gravity = g. JIPMER 2019 2
2S cosθ 2S 88 The lower end of a capillary tube of radius r is placed
(a) (b)
ρrg ρrg cos θ vertically in water. Then, with the rise of water in the
2S sin θ capillary, heat evolved is
(c) (d) None of these
ρrg π 2 r2 h 2 πr2 h 2 dg
(a) + dg (b)
2 2
85 A capillary tube A is dipped in water. Another
π 2 h 2 dg πr2 h 2 dg
identical tube B is dipped in soap-water solution. (c) − (d) −
2 2

SPECIAL TYPES QUESTIONS


I. Assertion and Reason 90 Assertion In steady flow, the velocity of each
■ Direction (Q. Nos. 89-105) In the following passing fluid particle remains constant in time.
questions, a statement of Assertion is followed by a Reason Each particle follows a smooth path and the
corresponding statement of Reason. Of the following paths of the particle do not cross each other.
statements, choose the correct one. 91 Assertion In streamline flow, A × v is constant.
(a) Both Assertion and Reason are correct and Reason Reason For incompressive flow, mass in = mass out.
is the correct explanation of Assertion.
92 Assertion The stream of water flowing at high speed
(b) Both Assertion and Reason are correct but Reason
is not the correct explanation of Assertion. from a garden horse pipe tend to spread like a
(c) Assertion is correct but Reason is incorrect. fountain when held vertically up, but tends to narrow
(d) Assertion is incorrect but Reason is correct. down when held vertically down.
Reason Speed of upstream decreases as its area of
89 Assertion Pressure is not a vector quantity.
cross-section increases and speed of downstream
Reason No direction can be assigned to pressure. increases as its area of cross-section decreases.
93 Assertion The steady flow of a liquid over a 104 Assertion Ploughing a field reduces evaporation of
horizontal surface in the form of layers of different water from the ground beneath.
velocity is called turbulent flow. Reason Ploughing results in lowering of surface area
Reason When a fluid is flowing in a pipe, then open to sunlight.
velocity of the liquid layer along the axis of tube is 105 Assertion Washing with water does not remove
maximum and decreases gradually as it move towards grease stains.
wall. Reason Water does not wet greasy dirt.
94 Assertion The shape of an automobile is so designed
that its front resembles the streamline pattern of the II. Statement Based Questions
fluid through which it moves. 106 I. Atoms or molecules in a fluid are arranged in a random
Reason The resistance offered by the fluid is not manner.
maximum. II. A fluid can withstand tangential or shearing stress for
95 Assertion The machine parts are jammed in winter. an indefinite period.
Reason The viscosity of the lubricants used in the III. A fluid has no definite shape of its own.
machine increases at low temperature. Which of the following statement (s) is/are correct?
96 Assertion Water flows faster than honey. (a) Only I (b) Both I and III
Reason The coefficient of viscosity of water is less (c) Only III (d) I, II and III
than honey.
107 In streamline flow, which of the following
97 Assertion All the rain drops hit the surface of the statement(s) is/are correct?
earth with the same constant velocity. I. Every fluid particle follows the path of its preceding
Reason An object falling through a viscous medium particle with same velocity.
eventually attains a terminal velocity. II. Eddies and whirls are formed.
98 Assertion No net force acts on a body falling in a III. Speed of particle is less than the critical speed.
liquid with a velocity equal to the terminal velocity. (a) Only I (b) Both I and III
Reason The weight of the body is balanced by the (c) Only III (d) Both II and III
upward buoyant force. 108 I. Turbulence dissipates kinetic energy in the form of
99 Assertion A fluid will stick to a solid surface. heat.
Reason Surface energy between fluid and solid is II. Turbulence like friction is sometimes desirable.
smaller than the sum of surface energies between III. Turbulence promotes mixing and increases the rate of
solid-air and liquid-air interface. transfer of mass, momentum and energy.
100 Assertion Sometimes insects can walk on the surface Which of the following statement(s) is/are correct?
of water. (a) Only I (b) Both II and III
(c) Only III (d) I, II and III
Reason The gravitational force on insect is balanced
by force due to surface tension. AIIMS 2019 109 I. Bernoulli’s equation ideally applies to non-viscous
fluids.
101 Assertion A bubble differs from a drop as it has two
interfaces. II. Fluid must be compressible in Bernoulli’s theorem.
Reason Excess pressure inside a drop is directly III. Bernoulli’s equation does not hold for non-steady flow.
proportional to its surface area. Which of the following statement(s) is/are correct?
102 Assertion The surface of water in the capillary tube (a) Both I and II (b) Only I
(c) Both I and III (d) Only II
is concave.
Reason The pressure difference between two sides of 110 For an aerofoil, which of the following are correct?
2S I. The orientation of the wing relative to flow direction
the top surface of water is cos θ. causes the streamlines to crowd above wing surface.
a
103 Assertion Smaller drop of water resist deformation II. Flow speed on top is lower than that below it.
forces better than the larger drops. III. Upward force results in a dynamic lift of the wings.
(a) Only I (b) Both I and II
Reason Excess pressure inside drop is inversely
(c) Only III (d) Both I and III
proportional to its radius. AIIMS 2018
111 I. Stress depends on the rate of change of strain or strain 116 An object is moving through a viscous fluid.
rate. Which of the following statement is incorrect?
II. The coefficient of viscosity for a fluid is defined as the (a) Viscous force on object decreases with increase in
ratio of shearing stress to the strain rate. temperature.
III. The SI unit of viscosity is Poiseuille (PI). (b) Viscous force depends on velocity of object.
Which of the following statement(s) is/are correct? (c) Viscous force decreases if object is made pointed.
(a) Only I (b) Both I and II (d) Viscous force is directed anti-parallel to velocity of
flow.
(c) Only II (d) I, II and III
117 A liquid is kept in a bowl opened to atmosphere.
112 I. Viscous force is proportional to the velocity of the object.
II. Viscous is a dragging force which acts opposite to the B A
direction of motion.
III. Viscous force depends on viscosity ηof the fluids and Consider two consecutive layers A and B as shown
radius r of the sphere.
above,
Which of following statement(s) is/are correct? which of the following statement(s) is/are correct?
(a) Only I (b) Both I and II
(a) Total energy of surface A = Total energy of surface B.
(c) Only III (d) I, II and III
(b) Number of molecules in surface A > Number of
113 A thin uniform cylindrical sheet, closed at both molecules in surface B.
ends,is partially filled with water. It is floating (c) Energy of a molecule of layer B is nearly half of energy
vertically in water in half-submerged state. of layer A.
If ρ c is the relative density of the material of the shell (d) Net force on a molecule of surface B is zero.
with respect to water, then the correct statement is 118 A small drop is formed using a dropper over a clean
that the shell is solid surface, then which of the following
(a) more than half filled, if ρc is less than 0.5. statement(s) is/are correct?
(b) more than half filled, if ρc is more than 1.0. Where, Sla = surface force of liquid and air,
(c) half filled, if ρc is more than 0.5.
S sa = surface force of solid and air
(d) less than half filled, if ρc is less than 0.5.
and S ls = surface force of liquid and solid.
114 A hydraulic lift has limbs of areas A1 and A2
(a) If S ls > S la , then angle of contact is less than 90°.
( A1 >> A2 ). A piston creates a force F2 in small limb (b) If S ls > S la , then angle of contact is equal to 90°.
to balance a large force F1 . Which of the following (c) If S ls > S la , then liquid spreads over solid surface.
statement is correct?
(d) If S ls > S la , then liquid does not spread on solid surface.
A1
(a) F2 ∝ .
A2 III. Matching Type
(b) Work done by F1 > Work done by F2 .
119 Match the Column I (terms or quantities) with
(c) Pressure in small area limb is more than pressure in
Column II (dimensions) and select the correct answer
large area limb.
from the codes given below.
(d) If piston of small area limb has velocity v2 and piston of
v A Column I Column II
large area limb as velocity v1 , then 2 = 1 .
v1 A2 A. Coefficient of viscosity 1. [ ML0T −2 ]
115 Which amongst the following statement is incorrect? B. Density 2. [ M0L0T 0 ]
(a) A jet of air striking a plate placed perpendicular to it is C. Surface tension 3. [ ML−1T −1 ]
an example of turbulent flow.
(b) The carburetor of automobile has a venturi channel D. Reynold’s number 4. [ ML−3T 0 ]
(nozzle) through which air flows with a high speed.
(c) Ball moving without spinning inside a fluid experiences A B C D A B C D
a net upward force. (a) 2 3 4 1 (b) 1 3 4 3
(d) None of the above (c) 3 4 1 2 (d) 1 2 3 4
120 Match the Column I (situation) with Column II 121 Match the Column I (situation) with Column II (value
(reason or principle) and select the correct answer or changes) and select the correct answer from the
from the codes given below. codes given below.
Column I Column II Column I Column II
A. Hydraulic lift 1. Archimedes’ principle A. When a single drop splits into 1. Zero
B. A razor blade can be made to 2. Pascal’s law n-identical drops, then
float on water surface in a tray. B. When a n-identical drops 2. Temperature
combine to form a single increase
C. The dam of water reservoir is 3. Surface tension
drop, then
made thick at the bottom level.
C. The surface tension of a liquid 3. Energy absorbed
D. Ship is floating on ocean water. 4. Pressure drop decreases, for
A B C D D. The surface tension of water 4. Energy released
at boiling temperature is
(a) 2 3 4 1
(b) 4 3 4 1 A B C D A B C D
(c) 4 1 3 4 (a) 3 4 1 2 (b) 3 4 2 1
(d) 1 2 3 4 (c) 4 3 1 2 (d) 1 2 3 4

NCERT & NCERT Exemplar


MULTIPLE CHOICE QUESTIONS
NCERT and 12.5 cm of spirit in the other, what is the specific
122 A 50 kg girl wearing high heel shoes balances on a gravity of spirit?
single heel. The heel is circular with the diameter (a) 0.6 (b) 0.7 (c) 0.8 (d) 0.9 m
1.0 cm. What is the pressure exerted on the horizontal 127 In a U-tube, mercury column in the two arms are in
floor? level, with 10 cm of water in one arm and 12.5 cm of
(a) 3 × 106 Pa (b) 2 × 104 Pa spirit in the other. If 15 cm of water and spirit each
(c) 6.24 × 106 Pa (d) 9 × 103 Pa are further poured into the respective arms of the
123 Toricelli’s barometer used mercury. Pascal duplicated tube, then what is the difference in the levels of
it using French wine of density 984. Determine the mercury in the two arms? (Specific gravity of
height of the wine column for normal atmospheric mercury =136 .)
pressure. (a) 0.106 cm (b) 0.221 cm (c) 0.302 cm (d) 0.136 cm
(a) 9 m (b) 10.5 m (c) 11.5 m (d) 7.5 m 128 Glycerin flows steadily through a horizontal tube of
124 A vertical off-shore structure is built to withstand a length 1.5 m and radius 1.0 cm. If the amount of
9
maximum stress of 10 Pa. What is the suitable glycerin flowing per second at one end is
pressure exerted by water column? Take, the depth of 4.0 ×10 −3 kg s −1 , then what is the pressure difference
the ocean to be roughly 3 km and ignore ocean current. between two ends of the tube? (Density of glycerin
(a) 4 × 104 Pa (b) 2.94 × 107 Pa = 1.3 × 10 3 kgm −3 and viscosity of glycerin
(c) 2.0 × 106 Pa (d) 3 × 105 Pa = 0.83 Pa-s).
125 A hydraulic automobile lift is designed to lift cars with a (a) 5 × 102 Pa (b) 9.75 × 102 Pa
maximum mass of 3000 kg. The area of cross-section of (c) 4 × 104 Pa (d) 2 × 102 Pa
the piston carrying the load is 425 cm 2 . What 129 In a test experiment on a model aeroplane in a wind
maximum pressure would the smaller piston can bear? tunnel, the flow speeds on the upper and lower
(a) 9 × 105 Pa (b) 6.92 × 105 Pa surfaces of the wing are 70 ms −1 and 63 ms −1 ,
(c) 5 × 105 Pa (d) 3 × 104 Pa respectively. What is the lift on the wing, if its area is
126 A U-tube contains water and methylated spirit 2.5 m 2 ? (Take, the density of air to be 1.3 kgm −3 )
separated by mercury. The mercury columns in the (a) 2 × 103 N (b) 4 × 102 N
two arms are in level with 10 cm of water in one arm (c) 1.51 × 103 N (d) 6 × 103 N
130 The cylindrical tube of a spray pump has a Give the absolute and gauge pressure of the gas in the
cross-section of 8.0 cm 2 , one end of which has enclosure for cases (a) and (b) in units of cm of
40 fine holes each of diameter 1.0 mm. If the liquid mercury.
flow inside the tube is 1.5 m/min, what is the speed of (a) 96 and 20, 58 and –18 cm
ejection of the liquid through the holes? (b) 84 and 10, 40 and –20 cm
(a) 0.94 ms −1 (b) 0.64 ms −1 (c) 0.25 ms −1 (d) 0.50 ms −1 (c) 30 and 20, 30 and 40 cm
131 A U-shaped wire is dipped in a soap solution and (d) 85 and 40, 20 and 40 cm
removed. The thin soap film formed between the wire 136 During blood transfusion, the needle is inserted in a
and a light slider supports a weight of 1.5 × 10 −2 N vein where the gauge pressure is 2000 Pa. At what
(which includes the small weight of the slider). The minimum height must the blood container be placed
length of the slider is 30 cm, then what is the surface so that blood may just enter the vein? The density of
tension of the film? whole blood is 1.06 × 10 3 kgm −3 .
(a) 2.5 × 10−2 Nm−1 (b) 5 × 10−3 Nm−1 (a) 0.6 m (b) 0.5 m
(c) 6 × 10−4 Nm−1 (d) 9 × 10−2 Nm−1 (c) 0.3 m (d) 0.2 m
132 The drop of mercury has radius 3.00 mm at room 137 (i) What is the largest average velocity of blood flow in
temperature. Surface tension of mercury at that an artery of radius2 × 10 −3 m, if the flow must
temperature is 4.65 × 10 −1 Nm −1 . The atmospheric remain laminar?
pressure is1.01 × 10 5 Pa, then what is the excess (ii) What is the corresponding flow rate?
pressure inside the drop at that temperature? (Take, viscosity of blood to be 2.084 × 10 −3 Pa-s and
(a) 410 Pa (b) 210 Pa (c) 540 Pa (d) 310 Pa
density of blood is 1.06 × 10 3 kgm −3 .)
133 What is the excess pressure inside a bubble of soap
solution of radius 5.00 mm, given that the surface (a) 0.98 ms −1 , 123
. × 10−5 m3s −1
tension of soap solution at the temperature 20° C is (b) 2 ms −1 , 123
. × 10−5 m3s −1
2.50 × 10 −2 Nm −1 ? If an air bubble of the same . ms −1 , 2 × 10−5 m3s −1
(c) 12
dimension were formed at a depth of 40.0 cm inside a (d) 3 ms −1 , 123
. × 10 m3s −1
container containing the soap solution (relative
density is 1.20), what would be the pressure inside the 138 A plane is in level flight at constant speed and each of
bubble? (1 atmospheric pressure is 1.01 × 10 5 Pa) its two wings has an area of 25 m 2 . If the speed of
(a) 10 Pa, 2 × 104 Pa (b) 20 Pa, 1.06 × 105 Pa the air is 180 kmh −1 over the lower wing and
(c) 20 Pa, 3 × 104 Pa (d) 30 Pa, 5 × 103 Pa 234 kmh −1 over the upper wing surface, determine
the mass of plane. (Take, air density to be 1 kgm −3 .)
134 A tank with a square base of area 1.0 m 2 is divided by
(a) 4000 kg (b) 5400 kg (c) 4400 kg (d) 5000 kg
a vertical partition in the middle. The bottom of the
partition has a small hinged door of area 20 cm 2 . The 139 In Millikan’s oil drop experiment, what is the
tank is filled with water in one compartment and an terminal speed of an uncharged drop of radius of
acid (of relative density 1.7) in the other, both to a 2.0 × 10 −5 m and density 1.2 × 10 3 kgm −3 ? Take the
height of 4.0 m. Compute the force necessary to keep viscosity of air at the temperature of the experiment to
the door closed. be 1.8 × 10 −5 Pa-s and how much is the viscous force
(a) 55 N (b) 27 N (c) 20 N (d) 60 N on the drop at that speed? Neglect buoyancy of the
135 A manometer reads the pressure of a gas in an drop due to air.
enclosure as shown in Fig. (a). When a pump removes (a) 3.9 × 10−3 ms −1 , 4 × 10−2 N
some of the gas, the manometer reads as in Fig. (b).
(b) 5.8 × 10−2 ms −1 , 3.93 × 10−10 N
The liquid used in the manometers is mercury and the
atmospheric pressure is 76 cm of mercury. (c) 2 × 10−3 ms −1 , 6 × 10−3 N
(d) 5.8 × 10−2 ms −1 , 7 × 10−4 N
To pump
20 cm

140 Mercury has an angle of contact equal to 140° with


sodalime glass. A narrow tube of radius 1.00 mm
18 cm

made of this glass is dipped in a trough containing


mercury.
By what amount does the mercury dip down in the
(a) (b)
tube relative to the liquid’s surface outside? Surface
278 Master The NCERT > PHYSICS (Vol-I )

tension of mercury at the temperature of the 145 An ideal fluid flows through a pipe of circular
experiment is 0.465 Nm −1 . (Density of mercury is cross-section made of two sections with diameters
13.6 × 10 3 kgm −3 and cos 140° = − 0.7660) 2.5 cm and 3.75 cm. The ratio of the velocities in the
(a) 2.34 mm (b) 4.34 mm (c) 5.34 mm (d) 6.34 mm
two pipes is
(a) 9 : 4 (b) 3 : 2
141 Two narrow bores of diametres 3.0 mm and 6.0 mm (c) 3 : 2 (d) 2 : 3
are joined together to form a U-tube opened at both
ends. If the U-tube contains water, what is the 146 The angle of contact at the interface of water-glass is
difference in its levels in the two limbs of the tube? 0°, ethyl alcohol-glass is 0°, mercury-glass is 140°
Surface tension of water at the temperature of the and methyliodide-glass is 30°. A glass capillary is put
experiment is 7.3 × 10 −2 Nm −2 . Take the angle of in a trough containing one of these four liquids. It is
contact to be zero, density of water to be observed that the meniscus is convex. The liquid in
1.0 ×10 3 kgm −3 and g = 9.8 ms −2 . the trough is
(a) 2.4 mm (b) 5.4 mm (c) 4.9 mm (d) 6.3 mm (a) water (b) ethylalcohol
(c) mercury (d) methyliodide

NCERT Exemplar 147 For a surface molecule,


(a) the net force on it is zero
142 A tall cylinder is filled with viscous oil. A round (b) there is a net downward force
pebble is dropped from the top with zero initial (c) the potential energy is less than that of a molecule inside
velocity. From the plot shown in figure, indicate the (d) the potential energy is equal to that of a molecule inside
one that represents the velocity v of the pebble as a
148 Pressure is a scalar quantity, because
function of time t .
(a) it is the ratio of force to area and both force and area are
vectors
(b) it is the ratio of the magnitude of the force to area
(a) v (b) v (c) it is the ratio of the component of the force parallel to
the area
t t
(d) it does not depend on the size of the area chosen
149 With increase in temperature, the viscosity of liquids
(a) decreases
(c) v (d) v (b) increases
(c) remains same
t t (d) None of the above
143 Which of the following diagram does not represent a 150 Streamline flow is more likely for liquids with
streamline flow? (a) high density (b) high viscosity
(c) low density (d) Both (b) and (c)
151 If a drop of liquid breaks into smaller droplets, it
(a) (b)
results in lowering temperature of the droplets. Let a
drop of radius R breaks into N small droplets each of
radius r. Estimate the drop in temperature.
3S  1 1 3 S  1 1
(a)  −  (b)  − 
ρ s  R 2 r2  ρ s  R r
(c) (d) 2
2S  1 1 4S  1 1
(c)  −  (d)  2 − 2
ρ s  R r 
ρs R r 

144 Along a streamline, 152 The surface tension and vapour pressure of water at
(a) the velocity of all fluid particles remains constant 20° C is 7.28 × 10 −2 Nm −1 and 2.33 × 10 3 Pa,
(b) the velocity of all fluid particles crossing a given respectively. What is the radius of the smallest
position is constant spherical water droplet which can be formed without
(c) the velocity of all fluid particles at a given instant is evaporating at 20°C?
constant (a) 5 × 10− 4 m (b) 6.25 × 10−5 m
(d) the speed of all fluid particles remains constant (c) 9 × 10−2 m (d) 3 × 10−5 m
Answers
>Mastering NCERT with MCQs
1 (d) 2 (d) 3 (a) 4 (a) 5 (a) 6 (c) 7 (d) 8 (a) 9 (a) 10 (b)
11 (a) 12 (c) 13 (d) 14 (d) 15 (d) 16 (a) 17 (d) 18 (b) 19 (c) 20 (a)
21 (d) 22 (b) 23 (a) 24 (b) 25 (c) 26 (b) 27 (d) 28 (a) 29 (b) 30 (a)
31 (c) 32 (c) 33 (b) 34 (c) 35 (a) 36 (d) 37 (b) 38 (c) 39 (c) 40 (d)
41 (d) 42 (b) 43 (b) 44 (d) 45 (b) 46 (a) 47 (d) 48 (a) 49 (c) 50 (b)
51 (a) 52 (a) 53 (b) 54 (b) 55 (d) 56 (d) 57 (b) 58 (b) 59 (a) 60 (d)
61 (a) 62 (d) 63 (a) 64 (b) 65 (a) 66 (a) 67 (d) 68 (d) 69 (a) 70 (b)
71 (a) 72 (a) 73 (d) 74 (b) 75 (a) 76 (b) 77 (b) 78 (c) 79 (b) 80 (b)
81 (c) 82 (b) 83 (b) 84 (a) 85 (d) 86 (c) 87 (a) 88 (b)

>Special Types Questions


89 (a) 90 (a) 91 (a) 92 (a) 93 (d) 94 (b) 95 (a) 96 (a) 97 (b) 98 (c)
99 (a) 100 (a) 101 (c) 102 (a) 103 (a) 104 (c) 105 (a) 106 (b) 107 (b) 108 (d)
109 (c) 110 (d) 111 (d) 112 (d) 113 (a) 114 (d) 115 (c) 116 (a) 117 (d) 118 (d)
119 (c) 120 (a) 121 (b)

>NCERT & NCERT Exemplar MCQs


122 (c) 123 (b) 124 (b) 125 (b) 126 (c) 127 (b) 128 (b) 129 (c) 130 (b) 131 (a)
132 (d) 133 (b) 134 (a) 135 (a) 136 (d) 137 (a) 138 (c) 139 (b) 140 (c) 141 (c)
142 (c) 143 (d) 144 (b) 145 (a) 146 (c) 147 (b) 148 (b) 149 (a) 150 (d) 151 (b)
152 (b)

Hints & Explanations


2 (d) The pointed ends of metal nails and metal pins have Total mass
6 (c) Density, ρ =
very small area. Total volume
When a force is applied over head of a pin or a nail, it 2m 2m
= =
transmits a large pressure (= force/area) on the surface V1 + V2 1 1
and hence easily penetrate the surface. m + 
 ρ1 ρ2 
4 (a) Total cross-sectional area of the femurs is
2ρ1 ρ2
A = 2 × 10 cm2 = 20 × 10−4 m2 . ∴ ρ=
ρ1 + ρ2
The force acting on them is F = 40 kg -wt = 400 N
(take, g = 10 ms −2 ). F
7 (d)Q Pressure at a point inside a liquid, p = = ρgh
A
This force is acting vertically downwards, so acts, ∴ It does not depend on the weight of fluid.
normally on the femurs.
9 (a) The forces on this element are those exerted by the
Thus, the average pressure is
rest of the fluid and they must be normal to the surfaces
F 400
pav = = = 2 × 105 Nm−2 of the element. Thus, the fluid exerts pressure pa , pb
A 20 × 10−4 and pc normal to the forces Fa , Fb and Fc .
Hints & Explanations

5 (a) Given, F1 : F2 = 1 : 7 and A1 : A2 = 3 : 2 Thus, in equilibrium, ΣFx = 0 and ΣF y = 0


F D
∴ Pressure, p = Fbsinθ
A A
θ Fb
p1 F1 / A1
∴ = Fc Fb cosθ
p2 F2 / A2 E
F A F
= 1 ⋅ 2 B C
F2 A1 Fa
1 2 2
= ⋅ = = 2: 21 ⇒ Fb sin θ = Fc and Fb cosθ = Fa
7 3 21
10 (b) Force acting on the base, F = p × A = hρgA In the given case,
= 0.4 × 900 × 10 × 2 × 10−3 = 7.2 N Angular speed, ω = 2rps = 2 × 2π = 4π rad s−1
Radius of vessel, r = 5 cm = 0.05 m
11 (a) Given, pressure = 150 mm of Hg = 015
. m of Hg
and g = 10 ms−2
ρ = 13.6 × 103 kg m −3 , g = 10 ms −2
Hence, substituting these values in Eq. (i), we get
. m,V = 5 × 10−3 m 3 and t = 60 s
h = 015 ω 2 r2 ( 4 π )2 ( 0.05 )2
dV 5 × 10−3 3 −1 y= = = 0.02 m = 2 cm
Pumping rate of heart of a man = = m s 2g 2 × 10
dt 60
14 (b) In the given situation as shown in the figure below
dV dV
Power of heart = p ⋅ = ρgh ⋅ (Q p = ρgh )
dt dt
(13.6 × 103 kg m−3 )(10 ) × (0.15 × 5 × 10−3 )
=
60 Water 20 cm
= 1.70 W 15 cm Oil

12 (c) Given, d = 2700 m and ρ = 103 kgm −3


Compressibility = 45.4 × 10−11 Pa −1
The pressure at the bottom of ocean is given by According to Pascal’s law,
p = ρgd = 103 × 10 × 2700 = 27 × 106 Pa Pressure due to water column of height 15 cm
= Pressure due to oil column of height 20 cm
So, fractional compression = compressibility × pressure
⇒ hwρw g = hoρo g
= 45.4 × 10−11 × 27 × 106 15
= 1.2 × 10−2 15ρw = 20ρo ⇒ ρo = ρω
20
15
13 (d) When liquid filled vessel is rotated the liquid profile ρo = × 1000 (Q given, ρw = 1000 kg m −3 )
becomes a paraboloid due to centripetal force, as shown 20
in the figure below = 750 kgm −3
y
15 (d) As we know, the liquid pressure is the same at all
r points at the same horizontal level.
P
Since in the given situation in the problem, both ends of
y the U-tube are open and the level of the fluid is same, so
the pressure on both the free surfaces must be equal.
O i.e. p1 = p2
x
hoil ⋅ ρoil ⋅ g = h water ⋅ ρ water ⋅ g (Q p = ρgh)
h water ⋅ ρ water ⋅ g
ρoil =
hoil ⋅ g
Pressure at any point P due to rotation is ( 65 + 65 ) × 1000
1 From figure, ρoil = = 928 kgm−3
pR = ρr2ω 2 ( 65 + 65 + 10 )
2
Gauge pressure at depth y is pG = − ρgy [Qρ water = 1000 kgm −3 ]
If p0 is atmospheric pressure, then total pressure at 16 (a) Pressure at left arm of U-tube,
point P is p1 = p0 + ρ1 gh1 = p0 + ρ1 10 (10 × 10−2 )
1 p1 = p0 + ρ1
p = p0 + ρr2ω 2 − ρgy or
Hints & Explanations

2 Pressure at right arm of U-tube,


For any point on surface of rotating fluid, p2 = p0 + ρ2 gh2 = p0 + ρ2 10 (12 × 10−2 )
p = p0
or p2 = p0 + 12. ρ2
Hence, for any surface point;
The mercury column in both arms of U-tube are at same
1
p0 = p0 + ρ r2ω 2 − ρ gy level, therefore pressure in both arms will be same.
2 i.e. p1 = p2 ⇒ p0 + ρ1 = p0 + 12 . ρ2
1 2 2
or ρr ω = ρgy ∴ Density of ethyl alcohol,
2 ρ 1000
ρ2 = 1 = = 833.3 kg/m 3
r2ω 2 12
. 12
.
or y= … (i)
2g = 0.83 gcm −3
17 (d) Mass per unit time of a liquid flow is given by (Atmospheric pressure is common to both pistons and
dm has been ignored.)
= ρAv
dt 21 (d) Given, mass of car = 500 kg
where, ρ is density of liquid, A is area through which it is Weight of car, w = mg = 500 × 10 = 5 × 103 N,
flowing and v is velocity. d1 = 2m ⇒ r1 = 1 m, d 2 = 20 cm
∴ Rate of change in momentum of the 25% of liquid ⇒ r2 = 10 cm = 0.1 m
which loses all momentum is
According to Pascal’s law,
dp1 1  dm 1
=   v = ρAv2 …(i) F2 F
dt 4  dt  4 = 1
A2 A1
and the rate of change in momentum of the 25% of the
F2 w
liquid which comes back with same speed. ⇒ =
dp2 1  dm 1 πr22 πr12
=   × 2v = ρAv2 …(ii)
dt 
4 dt  2 r 
2
.
 01
2
⇒ F2 = w  2  = 5 × 103   = 50 N
[Q Net change in velocity is = 2v]  r1   1
∴ Net pressure on the mesh is
23 (a) The buoyant force acting on the ice cube inside the
F ( dp1 / dt + dp2 / dt )  dp 
p = net = Q F = dt 
beaker depends on the value of g.
A A However, the fraction of the cube submerged under a
∴ From Eqs. (i) and (ii), we get liquid is independent of the value of g but depends only
3 3 on the density of the ice cube relative to that of the
p = ρv 2 A / A = ρv 2 liquid on which it floats. Therefore, if the beaker of
4 4
water is taken to the moon, where the gravity is (1 / 6 ) th
18 (b) As we know that, if p1 represents the atmospheric as that on earth, then the buoyant force acting on the
pressure, then absolute pressure p ′ = p1 + ρgh cube will be affected, but the fraction of the volume
and gauge pressure, p ′ ′ = p ′− p1 = ρgh submerged will remain same.
From the given figure, we can write, p1 = ρgh1 ∴On moon also, the ice cube floats on water in the
⇒ p ′ = p2 = ρgh1 + ρgh2 = ρg ( h1 + h2 ) = ρgh3  9
beaker with   th of its volume submerged under water.
 10
Similarly, p ′ ′ = p2 − p1 = ρgh3 − ρgh1
= ρ g ( h3 − h1 ) = ρ gh2 24 (b) According to the question, the situation can be drawn
∴ In terms of height, we can write, as
Absolute pressure = h3 = h1 + h2 l
Gauge pressure = h2 = h3 − h1 ρs
19 (c) Since, the fluid in the lift is perfectly incompressible.
ρm
∴ Volume covered by the movement of piston P1
inwards by a distance x1 is equal to volume moved Volume of block = l 3
outwards due to the piston P2 by a distance x2 (say).
∴ V1 = V2 ⇒ A1 x1 = A2 x2 Let h be the height of the block above the surface of
A Ax mercury and volume of mercury displaced = ( l − h ) ⋅ l 2 .
⇒ x2 = 1 x1 =
A2 2A ∴ Weight of mercury displaced = ( l − h ) ⋅ l 2 ⋅ ρm ⋅ g
(Given, A1 = A and A2 = 2 A) This is equal to the weight of the block which is
x ρs ⋅ l 3 ⋅ g
=
2
Thus, according to Archimedes’ principle,
x
Hints & Explanations

Thus, the distance moved by piston P2 is . ( l − h ) l 2 ⋅ ρm ⋅ g = ρs ⋅ l 3 ⋅ g


2
 ρ 
20 (a) Water is considered to be perfectly incompressible. which gives, h = l 1 − s 
Volume covered by the movement of smaller piston  ρm 
inwards is equal to volume moved outwards due to the
25 (c) According to question, the situation can be drawn as
larger piston.
following for a cylinder of area A.
L1 A1 = L2 A2
A π (1/ 2 × 10−2 )2 A
L2 = 1 L1 = × 6 × 10−2
A2 π ( 3/ 2 × 10−2 )2 (1– p)L d ρ

= 0.67 cm pL nρ
Applying Archimedes’ principle, 35 (a) From the Bernoulli’s theorem,
Weight of cylinder = ( Upthrust )1 + ( Upthrust ) 2 1
p1 − p2 = ρ ( v22 − v12 )
i.e. ALdg = (1 − p ) LAρg + ( pLA ) nρg 2
⇒ d = (1 − p ) ρ + pnρ = ρ − pρ + n pρ Given, . kgm −3
ρ = 13
d = ρ + ( n − 1) pρ = ρ {1 + ( n − 1) p} v2 = 120 ms −1
29 (b) Mass of fluid flowing out is equals the mass flowing
v1 = 90 ms −1
in, therefore ∆mP = ∆mQ = ∆mR , we have
AP vP ρP ∆t = AQ vQ ρQ ∆t = AR vR ρR ∆t 1
…(i) ⇒ p1 − p2 = × 1.3 × [(120 )2 − ( 902 )]
For flow of incompressible fluids, we have, 2
ρP = ρR = ρQ = 4095 Nm −2 or Pa
Thus, Eq. (i) reduces to, 1 2
36 (d) From Bernoulli’s equation, p = p0 + ρv
⇒ AP vP = AR vR = AQ vQ 2
Also, from the given figure we can also say that, Given, v = 40 ms −1 , A = 250 m 2 , ρair . kgm −3
= 12
AR > AQ > AP
Force will act due to pressure difference
So, at narrower portions, where the streamlines are
1 1
closely spaced, velocity increases. ∴ p − p0 = ρv2 = × 12 . × ( 40 )2
i.e. v p > vQ > vR 2 2
= 960 Pa
31 (c) Given, d1 = 6 cm ⇒ r1 = 3 cm
3 ∴ Force acting upwards, F
d 2 = 3 cm ⇒ r2 = cm = Pressure difference × Area
2
v = 2 ms −1 = 960 × 250 = 2.4 × 105 N, upwards
According to equation of continuity of flow, 37 (b) Given, diameter of tube at first end, d1 = 5 cm
A1 v1 = A2 v2 d1
Av π r2 v ∴ Radius, r1 = = 2.5 cm = 2.5 × 10−2 m
∴ v2 = 1 1 = 1 21 2
A2 πr2 Diameter of tube at second end, d 2 = 2 cm
(Q A1 = πr12 and A2 = πr22 ) d
2 2 ∴ Radius, r2 = 2 = 1 cm = 10−2 m
r  3  −1 2
= v1  1  = 2 ×   = 2 × 2 = 8 ms
2
 r2   3/ 2 Velocity of fluid at first end, v1 = 4 m/s
By the principle of continuity,
32 (c) If the liquid is incompressible, then according to
A1 v1 = A2 v2
Bernoulli’s principle mass of liquid entering through
left end, should be equal to mass of liquid coming out πr12 v1 = πr22 v2
from the right end. ( 2.5 × 10−2 )2 × 4 = (10−2 )2 ⋅ v2
∴ M = m1 + m2 ⇒ Av1 = Av2 + 15 . Av
⇒ v2 = 25 m/s
⇒ A × 3 = A × 15
. + 15. A v ⇒ v = 10 . ms −1 From Bernoulli’s theorem,
33 (b) Consider a cylindrical tube of a spray pump of 1 1
p1 + ρv12 = p2 + ρv22
radius R, one end having n fine holes, each of radius r 2 2
and speed of liquid in the tube is v as shown in figure. 1 2
p1 − p2 = ρ ( v2 − v12 )
2
r [Q density of water, ρ = 103 kg / m 3 ]
R
r 1
Hints & Explanations

= × 103 ( 252 − 4 2 )
2
According to equation of continuity, Av = constant = 304500 Pa
where, A is area of the cylindrical tube and v is velocity 38 (c) Applying Bernoulli’s theorem, at different parts of
of liquid in the tube. figure given below,
1 1
Volume of inflow rate = Volume of outflow rate pa + ρv12 + ρgy1 = p + ρv22 + ρgy2
2 2
vR 2
π R 2 v = n π r2 v ′ ⇒ v ′ = 2 = p + ρgy2 [Q v2 = 0]
nr
2
Thus, speed of the ejection of the liquid through the v1 = ( p − pa + ρg ( y2 − y1 ))
2
…(i)
ρ
vR 2
holes is 2 .
nr
To compressor
41 (d) The rate of liquid flow moving with velocity v
p v 2 =0 through an area a is given by
Rate ( R ) = Area ( A ) × Velocity ( v )
Given, area of hole, A = 2 mm 2 = 2 × 10−6 m 2
pa h
y2 Height of tank, h = 2 m
v
The given situation can also be depicted as shown in the
figure below
y1
Given, y2 − y1 = h
Substituting the given value in Eq. (i), we get 2m
2( p − pa ) 2(ρgh )
v1 = 2gh + = 2gh + = 2 gh
ρ ρ a=2mm2
39 (c) Let h be the depth of the hole below the free surface
of water. According to Torricelli’s theorem, the velocity As the velocity of liquid flow is given as v = 2gh
of the efflux v of water through the hole is given by ∴ R = Av = A 2gh
v = 2gh ...(i)
Substituting the given values, we get
The height through which the water falls is R = 2 × 10−6 × 2 × 10 × 2
s= H − h
If t is the time taken by water to strike the floor, then = 2 × 10−6 × 6.32 = 12.64 × 10−6 m 3 /s
1 1 ~ 12.6 × 10−6 m 3 /s

s = gt 2 or ( H − h ) = gt 2
2 2 42 (b) According to Torricelli’s theorem, the velocity of
⇒ t=
2( H − h )
…(ii) efflux of water, v = 2gh = 2 × 10 × 20 = 20 ms −1 .
g 43 (b) Given, ρ = 1000 kgm−3 , p1 = 3 × 105 Nm−2
and p2 = 1 × 105 Nm−2 (for air)
h Applying Bernoulli’s theorem, we get
1
H p1 + 0 + ρgH = p2 + ρv2 + ρgH
2

R
The distance R, where the emerging stream strikes the
floor is given by R = vt
H
Substituting the value of v and t from Eqs. (i) and (ii),
we get
2( H − h )
R = 2g ⋅ h × = 2 h(H − h ) 1 2
g p1 − p2 =ρv
2
40 (d) Given, height of small orifice from ground (H–h) 1
= 0.25 m 3 × 105 − 1 × 105 = ρv2
2
1 2
2 × 10 = ρv
5
h 2
Hints & Explanations

H 1
2 × 10 = × 103 × v2
5
2
0.25
v = 400
2

v = 400 ms −1
Total height of water tank, H = 1 m
∴ Range of water stream, 45 (b) Given, cross-sectional area of wider part of the
R = 2 h ( H − h ) = 2 ( H − 0.25 ) ( 0.25 ) meter, A = 8 mm2 , area of narrower part, a = 4 mm2
and density of blood, i.e. ρ is 1.06 × 103 kgm−3 .
= 2 (1 − 0.25 )0.25
A
= 2 0.75 × 0.25 The ratio of the areas is = 2.
a
= 0.866 m = 86.6 cm The pressure drop in artery = ρm gh = 24 Pa
So, the speed of the blood in the artery, 51 (a) For gases, viscosity increases with temperature.
−1 / 2
2ρm gh  A  2 Hence, coefficient of viscosity for hot air is greater than
v=   − 1 coefficient of viscosity for cold air.
ρ  a   52 (a) Most viscous liquid comes to rest quickly due to
2 × 24 −1 dissipation of energy at a larger rate.
⇒ v= −
~ 0.125 ms
. × 10 × (2 − 1)
106 3 2 Hence, most viscous liquid comes to rest at the earliest.
53 (b) Given, A = 0.10 m 2 , m = 0.010 kg, l = 0.30 mm
46 (a) According to Bernoulli’s principle, just before heart
attack as velocity increases due to constriction of artery, = 0.30 × 103 m and v = 0.085 ms −1 .
pressure is reduced and finally artery collapses.
The metal block moves to the right, because of the
47 (d) As the ball is not spinning, then by Bernoulli’s tension in the string. The tension T is equal to the
theorem, p A = pB . magnitude of the weight of the suspended mass m.
⇒ ( p A / pB ) = 1 Thus, the shear force,
48 (a) A ball which is spinning drags the fluid along with F = T = mg = 0.010 × 9.8 = 9.8 × 10− 2 N
it. The given figure shows the streamlines of fluid and
9.8 × 10− 2
spinning at the same time. Shear stress on the fluid = F / A =
The ball is moving forward and relative to it, the fluid is 0.10
v 0.085
moving backwards. Velocity gradient = =
l 0.30 × 10−3
Therefore, the velocity of the fluid above to the ball is
Stress
larger and below is smaller. This difference in the ⇒ Coefficient of viscosity, η =
velocity of fluid results in pressure difference between Velocity gradient
the lower and upper faces and there is a net upward (9.8 × 10−2 ) (0.30 × 10−3 )
force on the ball. =
(0.085 ) (0.10)
49 (c) The weight of the boeing aircraft is balanced by the
upward force due to the pressure difference. = 3.45 × 10−3 Pa-s
∆p × A = mg 54 (b) Given, r = 0.3 mm = 0.3 × 10−3 m, v = 1 ms −1
∆p × A = 3.3 × 105 × 9.8 η = 18 × 10−5 poise = 18 × 10−6 decapoise
∆p = (3.3 × 105 × 9.8 )/ 500 Viscous force, F = 6π ηrv
= 6.5 × 103 Nm−2 22
= 6× × (18 × 10−6 ) × ( 0.3 × 10−3 ) × 1
We ignore the small height difference between the top 7
and bottom sides in Bernoulli’s equation, = 1.018 × 10 −7 N
 1  1
i.e. p1 +   ρv12 + ρgh1 = p2 +   ρv22 + ρgh2 55 (d) According to Stoke’s law, the retarding force is
 2  2 proportional to velocity.
The pressure difference between them is Initially, when the spherical body is released in the
ρ fluid, it accelerates due to gravity. As the velocity
∆p = p1 − p2 = ( v22 − v12 )
2 increases, the retarding force also increases.
where, v2 is the speed of air over the upper surface and Finally, when viscous force plus buoyant force become
v1 is the speed under the bottom surface. equal to the force of gravity, the net force and hence
ρ acceleration become zero. The sphere then moves with a
∆p = ( v2 − v1 ) ( v2 + v1 ) constant velocity called terminal velocity. This situation
2
2∆p is correctly described by the v-t graph of option (d).
v2 − v1 = …(i) 56 (d) Final velocity is terminal velocity, it does not
ρ ( v2 + v1 )
Hints & Explanations

depend on the height of fall.


Taking the average speed,
57 (b) Given, vT = 6.5 × 10−2 ms −1 , a = 2 × 10−3 m
vav = ( v2 + v1 )/ 2 = 960 kmh −1 = 267 ms − 1
g = 9.8 ms −2 , ρ = 8.9 × 103 kgm−3
Dividing both sides of Eq. (i) by vav , we get
∆p σ = 1.5 × 103 kgm−3
v2 − v1 / vav = 2
ρvav 2a 2 (ρ − σ )g
So, terminal velocity, vT =
v − v1 9η
where, 2 represents the fractional increase in speed.
vav 2 (2 × 10−3 )2 × (8.9 − 1.5) × 103 × 9.8
⇒ η= × −2
v2 − v1 6.5 × 103 9 6.5 × 10
⇒ = ≈ 0.08
vav . × ( 267 )2
12 = 9.9 × 10−1 kg ms −1
2r2 (ρ − σ ) 61 (a) The rate of heat generation is equal to the rate of
58 (b) Terminal velocity is given by v = g
9η work done by the viscous force which in turn is equal to
its power.
where, r = radius of drop,
dQ
ρ = density of medium of drop, Rate of heat produced, = F × νT
σ = density of surrounding medium dt
and η = coefficient of viscosity of drop medium. where, F is the viscous force and vT is the terminal
⇒ v ∝ r2 and we know, area of drop ∝ r2 velocity.
⇒ v ∝ A (area) As, F = 6πηr ν T
v1 A 3 A 3 dQ
⇒ = 1 = ⇒ 1 = ⇒ = 6πηr ν T × ν T = 6πηr ν 2T …(i)
v2 A2 4 A2 4 dt
From the relation for terminal velocity,
59 (a) Terminal velocity is given by
2 r2 (ρ − σ )
2 r2 νT = g , we get
vT = (d − ρ )g 9 η

vP r2 ηQ ( d − ρP ) ν T ∝ r2 …(ii)
= P2 × ×
vQ rQ ηP ( d − ρQ ) From Eq. (ii), we can rewrite Eq. (i) as
dQ dQ
Given, d = 8 gcm −3 ∝ r ⋅ ( r2 )2 or ∝ r5
dt dt
0.5
rP = (1 / 2 ) cm, rQ = cm Hence, the rate of production of heat is proportional to r5 .
2
ρP = 0.8 gcm −3 64 (b) A liquid film has two surfaces, so upward force = 2Tl
. gcm −3
ρQ = 16 According to question,
Weight of the body hanged from wire ( mg )
ηP = 3 poise = Upward force due to surface tension ( 2Tl )
ηQ = 2 poise 2Tl
2 ⇒ m=
vP  1   2 ( 8 − 0.8 ) 2 7.2 g
⇒ =   ×  × = 4× × =3
vQ  0.5  3 ( 8 − 16
. ) 3 6.4
65 (a) Given, T = 70 × 10−3 Nm −1 , A = 10−2 m 2
⇒ vP : vQ = 3:1 and t = 0.05 mm = 0.05 × 103 m
60 (d) The terminal velocity achieved by ball in a viscous Force required to separate the two glass plates,
fluid is
2TA 2 × 70 × 10−3 × 10−2
2(ρ − σ )r2 g F = = = 28 N
vt = t 0.05 × 10−3

where, ρ = density of metal of ball, 66 (a) Given, F = 2 × 10−2 N and l = 10 cm = 10 × 10−2 m
σ = density of viscous medium, F 2 × 10−2
r = radius of ball Surface tension, T = = . Nm −1
= 01
2l 2 × 10 × 10−2
and η = coefficient of viscosity of medium.
Terminal velocity of first ball, 67 (d) Given, T1 = 0.07 Nm −1 m, T2 = 0.06 Nm −1
2(ρ1 − σ )r12 g 2 ( 8ρ2 − σ )r12 g and L = 2m
vt 1 = = …(i) Force on one side of the stick F1 = T1 × L
9η 9η
[Q ρ 1 = 8ρ2 ] = 0.07 × 2 = 014
. N
and force on other side of the stick
2 (ρ2 − σ )r22 g
Similarly, for second ball, vt 2 = …(ii) F2 = T2 × L = 0.06 × 2 = 012
. N
9η So, net force on the stick = F1 − F2 = 014
. − 012
.
Hints & Explanations

From Eqs. (i) and (ii), we get = 0.02 N


vt 1 2( 8ρ2 − σ )r12 g 9η 68 (d) Given, w = 1.5 × 10−2 N
= ×
vt 2 2(ρ2 − σ )r22 g 9η l = 30 cm = 30 × 10−2 m
2
 8ρ − 01. ρ2   r1  A liquid film has two free surfaces.
= 2    …(iii) [Qσ = 0.1ρ2 ]
 ρ2 − 01
. ρ2   r2  A slider will support the weight, F = 2Sl
when the force of surface tension
Given, r1 = 1 mm and r2 = 2 mm acting upwards on the slider ( = 2Sl )
Substituting these values in Eq. (iii), we get balances the downward force due to
vT1  7.9ρ2   1 2 79 weight ( w ) as shown below
⇒ =   = ∴ 2 Sl = w
vT 2
 0.9ρ2   2 36 ⇒ S = w / 2l
w
. × 10−2
15 74 (b) As, radius of bigger drop R = n1 / 3 r = 21 / 3 r
⇒ S =
2 × 30 × 10−2 ⇒ R 2 = 22 / 3 r2
∴ S = 0.025 Nm −1 r2
⇒ = 2−2 / 3
69 (a) A liquid air interface has energy. So, for a given R2
volume the surface with minimum energy is the one Initial surface energy 2( 4 π r2T )  r2 
with the least area. = = 2  2
Final surface energy ( 4 πR 2T ) R 
Since, amongst the various shapes of objects, sphere has
the minimum area. = 2 × 2−2 / 3 = 21 / 3 = 2 1 / 3 : 1
Thus, sphere (shape of drop and bubbles) will have 75 (a) Let n be the number of spherical drops of liquid of
minimum energy associated with it. radius r, that coalesce to form a single drop of radius R.
Therefore free liquid drops and bubbles are spherical in ∴ n × volume of each spherical drop of radius r =
shape, due to its surface with minimum energy, if volume of spherical drop of radius R
effects of gravity can be neglected.
V
70 (b) Suppose a spherical drop of radius r is in ⇒ nVi = V or Vi =
n
equilibrium. If its radius increases by ∆r, then the extra 4 3
surface energy is As, Vsphere = πr ⇒V ∝ r3
3
∆ES = final surface energy − initial surface energy
1 1 1
= ( SA ) f − ( SA )i As, V ∝ ⇒ r3 ∝ or r ∝ 1 / 3
n n n
where, S = surface tension and A is the surface area.
1
= | 4 π ( r + ∆r )2 − 4 πr2 | S ⇒ ri =
R
⇒ n3 =
R
1
= ( 4 πr + 4 πr + 8πr∆r − 4 πr ) S = 8πr ∆r S
2 2 2 r
n3
(neglecting ∆r2 as it is very small) As we know, ∆U = U f − U i = T 4 π ( R 2 − nr2 )
71 (a) If a liquid drop is in equilibrium, then energy lost is where, T is the surface tension of the liquid.
balanced by the energy gain due to expansion under the  
pressure difference ( pi − po ) between the inside of the 2 nr2  2  n
⇒ ∆U = T 4 πR 1 − 2  = T 4 πR 1 − 2 
 R 
drop and the outside.  n3 
Initial surface area of liquid drop = 4 πr2  R
= T × 4 πR 2 (1 − n1 / 3 ) = T 4 πR 2 1 − 
Final surface area of the liquid drop = 4 π ( r + ∆r )2  r
= 4 πr2 + 8πr∆r  1 1 4   1 1
= T 4 πR 3  −  = T 3  πR 3   − 
2
(∆r is very small and hence neglected)  R r 3   R r
Increase in the surface area of liquid drop  1 1
= 3 VT  − 
= 4 πr2 + 8πr∆r − 4 πr2 = 8πr∆r  R r
1 1
External work done is increasing the surface area of the As, R > r⇒ <
drop R r
w = 8πr∆r S la …(i)  1 1
∴ ∆U is negative, so energy released. = 3VT  – 
where, S la is the surface tension of liquid air interface.  r R
However, work done is W = ( pi − po ) 4 πr2 ∆r …(ii)
76 (b) A soap bubble is as shown in figure, differs from a
∴ From Eqs. (i) and (ii), we get drop and a cavity as it has two interfaces.
2S la po
pi − po =
Hints & Explanations

r
73 (d) Surface area of bubble of radius r = 4 πr2
pi
Surface area of bubble of radius 3r = 4 π ( 3r )2 = 36πr2 r
Therefore, increase in surface area
= 36πr2 − 4 πr2 = 32πr2 When radius of bubble is increased by radius ∆r, the
Since, a bubble has two surfaces, the total increase in increase in the surface area of the bubble = 8πr∆r.
surface area = 64 πr2 . So, effective increase in surface area of the soap bubble
= 2 × 8πr∆r = 16πr∆r
∴ Energy spent = Work done = Surface tension × Area
External work done in increasing the surface area of the
= 64 πσr2 soap bubble
= Increase in surface energy = 16πr∆r S la …(i) k  t −2 + 1  k −1 k 1
p=− . =− . = ⇒ p∝
where, S la is the surface tension of liquid-air interface. c  − 2 + 1 c t ct t
But, work done = p × 4 πr2 ∆r …(ii)
1
From Eqs. (i) and (ii), we get Hence, p versus graph is a straight line, which is
t
4 S la correctly represented in option (b).
p=
r 80 (b) The excess pressure inside a soap bubble of radius r
∴ Pressure difference in a soap bubble is is given by
4 S la
pi − p0 = p=
4T
r r
77 (b) Excess pressure inside an air bubble just below the where, T = surface tension.
2T
surface of water, p1 = , due to only one free surface If po be the pressure outside from the water, then total
r pressure inside the bubble becomes
2T
and excess pressure inside a drop, p2 = pi = po +
4T
… (i)
r r
∴ p1 = p2 The pressure at the depth Z0 below the water surface is
The excess pressure inside an air bubble below the p2 = po + Z0ρg … (ii)
surface of water is same as the excess pressure inside a As it is given that the pressure inside the bubble is same
drop of same radius outside the surface of water. as the pressure at depth Z0 , then equating Eqs. (i)
. 1 × 105 Pa, S = 7.30 × 10−2 Nm −1
78 (c) Given, p0 = 10 and (ii), we get
4T 4T
and r = 1mm = 1 × 10−3 m po + = po + Z0ρg ⇒ Z0 = … (iii)
r rρg
Pressure inside the bubble is
Given, T = 2.5 × 10 −2 N/m, ρ = 103 kg/m 3 ,
pi = po + 2S / r
= 1.01 × 105 + (2 × 7.30 × 10− 2 / 10–3 ) g = 10 m / s 2 and r = 1mm = 1 × 10−3 m
Substituting these values in Eq. (iii), we get
= (1.01000 + 0.00146) × 105 Pa = 1.01146 × 105 Pa
4 × 2.5 × 10−2
79 (b) When soap bubble is being inflated and its Z0 = = 10 × 10−3 m = 1cm
1 × 10−3 × 103 × 10
temperature remains constant, then it follows Boyle’s
law, so 81 (c) If po is the atmospheric pressure, the pressure
k outside the air bubble when it is at a depth
pV = constant (k) ⇒ p =
V h = po + hρg . Therefore, the total pressure inside the
Differentiating above equation with time, air bubble is
we get pt = p + po + hρg
d  1  −1 dV 2σ  2σ 
dp
= k.   ⇒
dp
= k 2 . = + po + hρg Q p = 
dt 
dt V  dt V  dt r  r
dV 82 (b) The excess pressure in a bubble of gas in a liquid is
It is given that, = c (a constant ) given by 2S/r, where S is the surface tension of the
dt
dp − kc liquid gas interface. There is only one liquid surface in
So, = 2 … (i) this case.
dt V
Pressure outside the bubble,
dV
Now, from = c; we get po = Atmospheric pressure + Pressure due to 8 cm of
dt water column
dV = cdt where, 1 atmospheric pressure = 101. × 105 Pa.
Hints & Explanations

or ∫ dV = ∫ cdt orV = ct … (ii) Pressure due to 8 cm of water column, p = ρgh


From Eqs. (i) and (ii), we get Density of water, ρ = 1000 kg/m3 , g = 9.8 ms −2
dp − kc dp  k h = depth below the surface of water in a beaker
= 2 2 or = −   t −2
dt c t dt  c = 8.00 cm = 0.08 m

k
dp = − . t −2 dt ∴ po = (1.01 × 105 Pa + 0.08 × 1000 × 9.8)
c = 1.01784 × 105 Pa
Integrating both sides, we get
k −2 83 (b) It is given that water rises to a height h in capillary
∫ dp = − c ∫ t dt tube. If the length of capillary tube above the surface
water is made less than h, then height of water column > 90 (a) The flow of the fluid is said to be steady, if at any
length of capillary tube. given point, the velocity of each passing fluid particle
So, water rises upto the top of capillary tube and stay remains constant in time.
there without overflowing. Every other particle which passes the second point
85 (d) Soap solution has lower surface tension, T as behaves exactly as the previous particle that has just
compared to pure water and capillary rise in tube, passed that point.
2T cos θ This is because, each particle follows a smooth path and
h= the paths of the particles do not cross each other.
ρrg
Therefore, Assertion and Reason are correct and Reason
so h is less for soap solution. is the correct explanation of Assertion.
So, figure in option (d) shows the correct relative nature
91 (a) According to the equation of continuity, for the flow
of liquid columns in the two tubes as water form
of incompressible fluids mass of liquid flowing out
concave surface with capillary tube.
equals to the mass of the liquid flowing in, i.e. Av =
86 (c) Given, h1 = 2.2 cm and h2 = 6.6 cm constant.
1 So, Av gives the volume flux or flow rate and remains
As, h ∝ constant throughout the streamline flow.
r
h1 r2 r h 6.6 3 Therefore, Assertion and Reason are correct and Reason
∴ = or 1 = 2 = =
h2 r1 r2 h1 2.2 1 is the correct explanation of Assertion.
92 (a) As from equation of continuity, volume flow rate of
87 (a) Height of liquid rise in capillary tube,
fluid, i.e. Q = Av = constant. So, speed of upstream
2T cosθ c 1
h= ⇒ h∝ decreases as its area of cross-section increases. Similarly,
ρrg r speed of downstream increase, as its area of cross-section
So, when radius is doubled, height becomes half. decreases.
∴ h′ = h / 2 So, when a horse pipe is held vertically up, the speed of
stream decreases and hence area of liquid flow increases
mass( M )
Now, density (ρ) = and water spread like a fountain. Similarly, when it is
volume(V ) held vertically down, the speed of stream increases , so
⇒ M = ρ × π r2 h area of liquid flow decreases and the water stream tends
to narrow down.
∴ M ′ = ρπr′ 2 h ′
Therefore, Assertion and Reason are correct and Reason
M ′ r′ 2 h ′ ( 2r )2 ( h / 2 )
∴ = 2 = =2 is the correct explanation of Assertion.
M r h r2 h 93 (d) In steady flow of a liquid over a horizontal surface,
⇒ M ′ = 2M
the velocities of layers increases uniformly from bottom
88 (b) When the tube is placed vertically in water, water (zero velocity) to the top layer (velocity v ).
2T cos θ
rises through a height h is given by h = For any layer of liquid, its upper layer pulls it forward
rdg while lower layer pulls it backward. This results in a
However, the increase in potential energy ∆EP , of the force between the layers. This type of flow is known as
h laminar flow.
raised water column = mg …(i) When a fluid is a flowing in a pipe or a tube, then
2
velocity of the liquid layer along the axis of the tube is
where, m is the mass of the raised column of water, maximum and decreases gradually as we move towards
i.e. m = πr2 hd the walls where it becomes zero as shown in figure.
From Eq. (i), we get
 hg  πr h dg
2 2
∆E p = ( πr2 hd )   =
 2 2
Hints & Explanations

Further, heat evolved = increase in potential energy


πr2 h 2 dg
∆W = ∆E p =
2 Therefore, Assertion is incorrect but Reason is correct.
89 (a) Pressure exerted is same in all directions in a fluid 94 (b) When a body moves through a fluid, its motion is
at rest. So, pressure is not a vector quantity. opposed by the force of fluid friction called resistance
No direction can be assigned to pressure. The force of fluid. It acts normal to the surface and increases with
against any area within (or bounding) a fluid at rest and increasing speed of body.
under pressure is normal to the area, regardless of the It is due to this reason, the shape of an automobile is, so
orientation of the area. designed that it resembles the streamline pattern of the
Therefore, Assertion and Reason are correct and Reason fluid through which it moves, so that air friction is
is the correct explanation of Assertion. minimum.
Also, the resistance offered by the fluid is not maximum. the pressure of water inside the tube, just at the
Therefore, Assertion and Reason are correct but Reason meniscus (air-water interface) is less than the
is not the correct explanation of Assertion. atmospheric pressure. So, the surface of water in the
capillary is concave as shown below
95 (a) The machine parts are jammed in winter
θ r
because the viscosity of the lubricants used in the
machines increases at low temperature.
Therefore, Assertion and Reason are correct and Reason
is the correct explanation of Assertion. θ
96 (a) Water flows faster than honey because the
coefficient of viscosity of water is less
than honey.
Therefore, Assertion and Reason are correct and Reason Therefore, Assertion and Reason are correct and Reason
is the correct explanation of Assertion. is the correct explanation of Assertion.
97 (b) When an object falls through a viscous medium, 103 (a) As excess pressure inside a liquid drop is
finally it attains terminal velocity. At this velocity, the 2S 1
viscous force balances the weight of the rain drop. p= ⇒ p∝
R R
So, all the rain drops hit the surface of the earth with the ∴ Excess pressure inside the smaller drop is large due to
same constant velocity, but do not mutually attain same which smaller drop of water resist deforming forces
terminal velocity because it depends on the size of the better than the larger drops.
drop. Therefore, Assertion and Reason are correct and Reason
Therefore, Assertion and Reason are correct but Reason is the correct explanation of Assertion.
is not the correct explanation of Assertion.
104 (c) The formation of capillaries take place in the field
98 (c) The weight of the body is balanced by two upward which is not ploughed for long. So, the water from
forces, namely the buoyant force and viscous force.
beneath the ground reaches the surface and evaporates.
No net force acts on a body falling in a liquid with a
But if the fields are ploughed, the capillaries will break
velocity equal to the terminal velocity, because this
and water will not rise to surface and thus ploughing
force (viscous) is balanced by the weight of body.
reduces evaporation.
Therefore, Assertion is correct but Reason is incorrect. On ploughing, more surface area of the field becomes
99 (a) A fluid will stick to a solid surface, open to the sunlight.
if the surface energy between fluid and the solid is Therefore, Assertion is correct but Reason is incorrect.
smaller than the sum of surface energies between
105 (a) Washing with water does not remove grease stains.
solid-air and liquid-air, interface.
This is because water does not wet greasy dirt, i.e. there
Therefore, Assertion and Reason are correct and Reason is very little area of contact between them.
is the correct explanation of Assertion.
Therefore, Assertion and Reason are correct and Reason
100 (a) Sometimes insects can walk on the surface of water is the correct explanation of Assertion.
due to surface tension, when legs of insects are not
being wet. 106 (b) Statements I and III are correct but II is incorrect
and it can be corrected as,
In this situation, the gravitational force on insect is
balanced by force due to surface tension. A fluid cannot withstand tangential or shearing stress
for an indefinite period. It begins to flow when a
Therefore, Assertion and Reason are correct and Reason shearing stress is applied.
is the correct explanation of Assertion.
107 (b) Statements I and III are correct but II is incorrect
101 (c) The excess pressure inside a liquid drop
and it can be corrected as,

is given by p = Eddies and whirls are formed in turbulent flow.
R
Hints & Explanations

1 109 (c) Statements I and III are correct but II is incorrect


i.e. p∝ and it can be corrected as,
R
A bubble differs from a drop as it has two interfaces. A restriction on application of Bernoulli’s theorem is
that the fluids must be incompressible as the elastic
Therefore, Assertion is correct but Reason is incorrect.
energy of the fluid is also not taken into consideration.
102 (a) The pressure difference between the two sides of the
110 (d) Statements I and III are correct but II is incorrect
top surface of a liquid in a capillary tube is given by
2S 2S 2S and it can be corrected as,
pi − po = = = cos θ The flow speed on top is higher than that below it.
r a secθ a
where, θ = angle of contact. 113 (a) Let V1 = total volume of material of shell
In case of water is taken in the capillary tube, the V2 = total inside volume of shell,
contact angle between water and glass is acute. Thus,
and x = fraction of V2 volume filled with water. Thus, viscous force on object decreases with decrease in
In floating condition, temperature.
Total weight = Upthrust Rest statements are correct.
V + V2  117 (d) From the property of surface tension, there are least
∴ V1ρc g + ( xV2 )(1)g =  1  (1)g
 2  number of molecules in topmost layer of any liquid and
[As upthrust is on half part only] for topmost layers of a fluid, energy is maximum.
[Qρ water = 1kgm −3 ] So, total energy of surface A > total energy of surface B.
V Number of molecules on surface A < Number of
⇒ x = 0.5 + ( 0.5 − ρc ) 1 molecules in surface B.
V2
The molecule on the surface of the liquid, i.e. layer A is
From here, we can see that, x > 0.5 if ρc < 0.5. surrounded from half side by liquid molecules. Thus, its
Thus, the statement given in option (a) is correct, rest potential energy is half that of a molecule inside the
are incorrect. liquid, i.e. layer B.
114 (d) Since, pressure is transmitted undiminished However, as the liquid molecule inside the liquid is
throughout the fluid in lift, surrounded equally from all sides, so net force on a
So, pressure in limb of area A1 = pressure in limb of molecule of surface B will be zero.
area A2 Thus, the statement given in option (d) is correct, rest are
F1 F2 A2 incorrect..
⇒ = ⇒ F2 = F1 ⇒ F2 ∝ A2 / A1
A1 A2 A1 118 (d) According to question, the situation can be depicted as
below,
Since, the fluid used is considered to be perfectly
incompressible.
Volume displaced by pistons in both limbs is same Sla
∴ V1 = V2 θ
⇒ A1 x1 = A2 x2
d d Ssa Sls
⇒ A1 ⋅ x1 = A2 x2
dt dt From, this we have,
⇒ A1 v1 = A2 v2 cos θ + S ls = S sa
v2 A
⇒ = 1 It is given that a small drop is formed, so the angle of
v1 A2 contact should be greater than 90°, i.e. θ > 90° and cos θ
As pressure is same in both the limbs, so work done is negative.
by force F1 is equal to that of F2 . This implies that S ls > S la . Hence, liquid does not spread
Thus, the statement given in option (d) is correct, rest on solid surface.
are incorrect. Thus, the statement given in option (d) is correct, rest are
115 (c) Statement given in option (c) is incorrect and it can incorrect.
be corrected as, 122 (c) Given, mass of girl, m = 50 kg
The streamlines around a non-spinning ball moving
Diameter of circular heel, 2r = 1.0 cm
relative to a fluid is as shown below.
∴ Radius, r = 0.5 cm = 5 × 10−3 m
Area of circular heel, A = πr2 = 3.14 × (5 × 10−3 )2 m2
= 78.50 × 10−6 m2
∴ Pressure exerted on the horizontal floor,
F mg 50 × 9.8
p= = =
Hints & Explanations

A A 78.50 × 10−6
From the symmetry of streamlines, it is clear that the
= 6.24 × 106 Pa
velocity of fluid above and below the ball at
corresponding points is the same resulting in zero 123 (b) Pressure exerted by h height of wine column ( hρg )
pressure difference. The fluid therefore, exerts no = Pressure exerted by 76 cm of Hg column ( hρg )
upward or downward force on the ball.
or h × 984 × 9.8 = 0.76 × 13.6 × 103 × 9.8
Rest statements are correct.
0.76 × 13.6 × 103
116 (a) Statement given in option (a) is incorrect and it ∴ h= = 10.5 m
can be corrected as, 984
Viscous force decreases with decrease in viscosity. If 124 (b) Given, depth of ocean, h = 3 km = 3000 m
fluid is a gas, its viscosity increases with increase in
Density of water, ρ = 103 kgm−3
temperature.
Pressure exerted by water column As the pressure in water arm is more, so mercury will
p = hρg = 3000 × 103 × 9.8 rise in spirit arm.
If this pressure difference corresponds to height
= 29.4 × 106 Pa = 2.94 × 107 Pa
difference h in the two arms as shown in figure, then
125 (b) Given, maximum mass that can be lifted, p1 − p2 = hρg
m = 3000 kg 3
Area of cross-section, A = 425 cm2 = 4.25 × 10−2 m2 25g − 22g = h × 13.6 × g or h = = 0.221 cm
13.6
∴ Maximum pressure on the bigger piston, Thus, mercury rises in the arm containing spirit and the
F mg difference in the levels of mercury in the two columns is
p= =
A A 0.221 cm.
3000 × 9.8 128 (b) Volume of the liquid flowing per second,
= = 6.92 × 105 Pa
4.25 × 10−2 Mass collected per second 4.0 × 10−3 3 −1
V = = ms
According to Pascal’s law, the pressure applied on an Density . × 103
13
enclosed liquid is transmitted equally in all directions.
∴ Maximum pressure on smaller piston = Maximum πpr4
But, V =
pressure on bigger piston 8l η
p ′ = p = 6.92 × 105 Pa 8lηV 8 × 15
. × 0.83 4.0 × 10−3
∴ p= = −2 4
×
126 (c) As, the mercury columns in the two arms of the πr 4
. × (1.0 × 10 )
314 . × 103
13
U-tube are at the same level, as shown below therefore = 9.75 × 102 Pa
Pressure due to water column = Pressure due to spirit
column 129 (c) Let the lower and upper surface of the wing of the
hw ρw g = hs ρs g or hw ρw = hs ρs aeroplane be at the same height h and speeds of air on
the upper and lower surfaces of the wing be v1 and v2 ,
Spirit respectively.
Speed of air on the upper surface of the wing,
v1 = 70 ms −1
12.5 cm

Water
10 cm

Speed of air on the lower surface of the wing,


v2 = 63 ms −1
A
Mercury
B Density of the air, ρ = 1.3 kgm−3
Area, A = 2.5 m2
Given, hw = 10 cm According to Bernoulli’s theorem,
ρw = 1 gcm−3 1 1
p1 + ρv12 + ρgh = p2 + ρv22 + ρgh
hs = 12.5 cm 2 2
∴ 10 × 1 = 12.5 × ρs 1
10 ⇒ p2 − p1 = ρ ( v12 − v22 )
or ρs = = 0.8 gcm−3 2
12.5
∴ Lifting force acting on the wings,
ρ 0.8 gcm−3
Specific gravity of spirit = s = = 0.8 1
F = ( p2 − p1 ) × A = ρ( v12 − v22 ) × A
ρw 1gcm−3 2
127 (b) Pressure on mercury level in one arm due to water,  Force
Q Pressure = 
p1 = hw ρw g  Area 
= (10 + 15 ) × 1 × g = 25 g 1
Hints & Explanations

= × 1. 3 × [(70)2 − (63)2 ] × 2.5


Spirit 2
1
= × 1.3 [4900 − 3969] × 2.5
27.5 cm

Water 2
25 cm

1
= × 1.3 × 931 × 2.5 = 1.51 × 103 N
2
Mercury h The pressure difference between two ends of the tube is
A 1.51 × 103 N.
130 (b) Area of cross-section of tube,
Pressure on mercury level in another arm due to spirit, A = 8.0 cm2 = 8.0 × 10− 4 m2
p2 = hs ρs g = (12.5 + 15 ) × 0.8 × g = 22 g
Number of holes, N = 40
Diameter of each hole, 2r = 1.0 mm Total pressure inside air bubble = Atmospheric pressure
Cross-section area, A1 = 8.0 cm 2 + Pressure due to 40 cm soap solution + Excess pressure
. × 105 + hρg + ρ′
= 101
∴ Radius of each hole, r = 0.5 mm = 5 × 10− 4 m
= 1.01 × 105 + 0.40 × 1200 × 9.8 + 10
1.5 −1
Velocity of liquid flow in tube = 1.5 m/ min = ms = 1.06 × 105 Pa
60
= 2.5 × 10− 2 ms −1 134 (a) For compartment containing water
Height of water column, h = 4.0 m
Total area of holes, A2 = N × πr 2

Density of water, ρ = 103 kgm−3


= 40 × 3.14 × (5 × 10− 4 )2
Pressure due to water at the door at the bottom,
= 3.14 × 10− 5 m2
pw = hρ g = 4.0 × 103 × 9.8
According to equation of continuity, A 1 v1 = A 2 v2
= 39.2 × 103 Pa
So, speed of ejection of the liquid through the hole
A 1 v1 8.0 × 10− 4 × 2.5 × 10− 2 Water Acid
v2 = = −5
A2 3.14 × 10
20
= × 10− 1 = 0.64 ms −1

4.0 m
3.14
131 (a) Given, F = 1.5 × 10−2 N and l = 30 cm = 0.3 m
Door
As the soap film has two free surfaces, so the force F
acts over the twice the length of the slider. For compartment containing acid
Hence, surface tension (S), Height of acid column, h = 4.0 m
F 1.5 × 10−2 Density of acid, ρ = relative density × 103
S = = = 2.5 × 10−2 Nm−1
2l 2 × 0.30 = 1.7 × 103 kgm−3
132 (d) Excess pressure inside a liquid drop is given by Pressure due to acid at the door at the bottom,
2S
∆p = , where S = surface tension of the liquid and pa = hρ g = 4.0 × 17. × 103 × 9.8
R
R = radius of the drop. = 66.64 × 103 Pa
Given, radius of drop, R = 3.00 mm = 3.00 × 10− 3 m ∴ pa − pw = 66.64 × 103 − 39.2 × 103
−1 −1
Surface tension of mercury, S = 4.65 × 10 Nm = 27.44 × 103 Pa
Atmospheric pressure, po = 1.01 × 105 Pa Area of the door, A = 20 cm2 = 20 × 10−4 m2
Excess pressure inside the drop,
Force on the door due to difference of pressure on its two
2S 2 × 4.65 × 10−1 sides
∆p = =
R 3.00 × 10−3 = ( pa − p w ) × A
= 3.10 × 102 = 27.44 × 103 × 20 × 10−4
= 310 Pa = 54.88 N ≈ 55 N
133 (b) Given, σ = 2.50 × 10−2 Nm −1 , R = 5.00 mm 135 (a) Atmospheric pressure, p0 = 76 cm of mercury
According to Fig. (a) given in question pressure head,
= 5.00 × 10−3 m,
h1 = 20 cm of mercury
Relative density = 120
. , h = 40 cm = 0.40 m ∴ Absolute pressure of the gas, p = p0 + h1ρg
Hints & Explanations

Excess pressure inside a soap bubble, = 76 cm of Hg + 20 cm of Hg


4σ 4 × 2.50 × 10−2 = 96 cm of Hg
p= = = 20 Pa
R 5.00 × 10−3 Gauge pressure = Absolute pressure
Excess pressure inside an air bubble under soap − Atmospheric pressure
2σ 2 × 2.50 × 10−2 = 96 cm of Hg − 76 cm of Hg = 20 cm of Hg
solution, p ′ = = = 10 Pa According to Fig. (b) given in question pressure head,
R 5.00 × 10−3
h2 = − 18 cm of mercury
Density of soap solution, ∴ Absolute pressure of the gas, p ′ = p0 + h2ρg
ρ = Relative density × 1000 = 76 cm of Hg + ( − 18 cm of Hg)
. × 1000 = 1200 kgm−3
= 120 = 58 cm of Hg
Gauge pressure = Absolute pressure 2 r2 (ρ − ρ0 ) g
Terminal velocity, v =
− Atmospheric pressure 9 η
= 58 cm of Hg − 76 cm of Hg 2 (2.0 × 10−5 )2 × (1.2 × 103 − 0) × 9.8
= ×
= − 18 cm of Hg 9 1.8 × 10− 5
. × 103 kgm −3
136 (d) Given, pg = 2000 Pa and ρ = 106 −2
= 5.8 × 10 ms −1

Let h be the height of container at which its blood exerts Viscous force acting on the drop
pressure equal to gauge pressure in vein, then (according to Stoke’s law), F = 6 πηrv
= 6 × 3.14 × 1.8 × 10−5 × 2.0 × 10−5 × 5.8 × 10−2
hρg = pg
pg = 3.93 × 10− 10 N
2000
⇒ h= =
ρg 106 . × 103 × 9.8 140 (c) Given, angle of contact, θ = 140°
= 01925
. m Radius of tube, r = 1 mm = 1 × 10−3 m
The blood will just enter the vein, if the blood container Surface tension, S = 0.465 Nm−1
is kept at height slightly greater than 0.1925 m, i.e. at Density of mercury, ρ = 13.6 × 103 kgm−3
0.2 m.
Height of liquid rise or fall due to surface tension,
137 (a) (i) Given, ρ = 1.06 × 103 kgm−3 2 S cos θ 2 × 0.465 × cos 140°
h= =
D = 2r = 4 × 10−3 m rρg 1 × 10− 3 × 13.6 × 103 × 9.8
and η = 2.084 × 10−3 Pa -s 2 × 0.465 × ( − 0.7660)
=
The maximum value of Reynold’s number for laminar 10− 3 × 13.6 × 103 × 9.8
flow is 2000. Hence, the maximum average velocity for
= − 5.34 × 10−3 m = − 5.34 mm
laminar flow or critical velocity is given by
Hence, the mercury level will depressed by 5.34 mm.
R η 2000 × 2.084 × 10−3
vc = e = d1 3.0
ρ ⋅ D 1.06 × 103 × 4 × 10−3 141 (c) Given, r1 = = . mm = 1.5 × 10−3 m,
= 15
2 2
= 0.98 ms −1
d 6.0
(ii) Volume of blood flowing per second, r2 = 2 = = 3 mm = 3 × 10−3 m,
2 2
V = avc = πr2 vc S = 7.3 × 10−2 Nm −2
22
= × ( 2 × 10−3 )2 × 0.98 . × 103 kgm −3
ρ = 10
7
g = 9.8 ms −2
= 1.23 × 10−5 m3s −1
Let h1 and h2 be heights to which water rise in the two
138 (c) Given, v1 = 180 kmh −1 = 50 ms −1 tubes, then
v2 = 234 kmh −1 = 65 ms −1 2S cosθ 2S cosθ
h1 = and h2 =
Area of the wings, A = 2 × 25 = 50 m2 , ρ = 1 kgm−3 r1 ρg r2 ρg
For a plane in the level flight, Bernoulli’s equation is 2S cosθ  1 1 
1 1 Therefore, h1 − h2 =  − 
p1 + ρv12 = p2 + ρv22 ρg  r1 r2 
2 2
1 2 2 × 7.3 × 10−2 × cos 0°  1 1 
p1 − p2 = ρ ( v2 − v12 ) =  − 
2 . × 10 × 9.8
10 3  1.5 × 10−3 3 × 10−3 
1
= × 1 × ( 652 − 502 ) = 862.5 Nm−2 = 0.49 × 10−2 m = 4.9 mm
2
142 (c) When the pebble is falling through the viscous oil,
Hints & Explanations

Upward force on the plane = ( p1 − p2 ) × A the viscous force is F = 6πηrv


= 862.5 × 50 = 43125 N
where, r is radius of the pebble, v is instantaneous speed
In level flight, the upward force balances the weight of
and η is coefficient of viscosity. As the force is variable,
the plane, so
hence acceleration is also variable, so v-t graph will not
mg = 43125 N be straight line. First velocity increases due to gravity
43125
∴ Mass of the plane, m = = 4400 kg and then becomes constant known as terminal velocity.
9.8 143 (d) In a streamline flow at any given point, the velocity
139 (b) Given, radius of drop, r = 2.0 × 10− 5 m of each passing fluid particles remains constant. If we
−3 consider a cross-sectional area, then a point on the area
Density of oil, ρ = 1.2 × 10 kgm
3
cannot have different velocities at the same time, hence
Viscosity of air, η = 1.8 × 10− 5 Pa -s two streamlines of flow cannot cross each other.
144 (b) As we know for a streamline flow of a liquid, 148 (b) Pressure is defined as the ratio of magnitude of
velocity of each particle at a particular position is component of the force normal to the area and the area
constant, because Av = constant (law of continuity) under consideration. As magnitude of component is
between two cross-section of a tube of flow. considered, hence it will not have any direction.
145 (a) Consider the diagram where an ideal fluid is flowing So, pressure is a scalar quantity.
through a pipe.
v1 v2 151 (b) When a big drop of radius R breaks into N droplets
d2 each of radius r, then in volume remains constant.
∴ Volume of big drop = N × Volume of small drop
d1
4 4
π R 3 = N × π r3
3 3
A1 A2
⇒ R = Nr
3 3
Given, d1 = diameter at 1st point is 2.5 cm
d 2 = diameter at 2nd point is 3.75 cm R3
⇒ N =
Applying equation of continuity for cross-sections A1 and r3
A2 . Now, change in surface area ,∆A = 4 πR 2 − N 4 πr2
⇒ A1 v1 = A2 v2 = 4 π ( R 2 − Nr2 )
2
v1 A π ( r22 ) r  Energy released = S × ∆A
⇒ = 2 = =  2
v2 A1 π ( r12 )  r1  = S × 4 π ( R 2 − Nr2 )
2 Due to releasing of this energy, the temperature is
 3.75   d 
  2 Q r2 = 2 
 lowered.
 3.75 9
= 2  =  =  2
If ρ is the density and s is specific heat of liquid and its
 2.5   2.5  4 and = d1 
 temperature is lowered by ∆θ, then
 2  2
1
Energy released = ms∆θ
146 (c) According to the question, the observed meniscus is 4 
of convex shape which is only possible when angle of S × 4 π ( R 2 − Nr2 ) =  πR 3 × ρ s ∆θ
3 
contact is obtuse as shown below. Hence, the
combination will be of mercury-glass is 140°. S × 4 π ( R 2 − Nr2 )
∆θ =
4
πR 3 ρ × s
Convex 3
3 S  R 2 Nr2 
140° =  − 3
ρs  R3 R 

Mercury 3 S  1 ( R 3 / r3 ) × r2 
=  − 
ρs  R R3 
3 S  1 1
147 (b) Consider the diagram shown below, where two =  − 
molecules of a liquid are shown. One is well inside the ρ s  R r
liquid and other is on the surface. The molecule A
152 (b) Given, surface tension of water,
which is well inside experiences equal forces from all
directions, hence net force on it will be zero. S = 7.28 × 10− 2 Nm−1
And molecules on the liquid’s surface have some extra Vapour pressure, p = 2.33 × 103 Pa
energy, as it is surrounded by liquid molecules only from The drop will evaporate, if the water pressure is greater
lower half side. than the vapour pressure. Let a water droplet of radius R
Hence, for a surface molecule, there is a net downward can be formed without evaporating.
force. ∴ Vapour pressure = Excess pressure in drop
2S
B p=
R
2S 2 × 7.28 × 10− 2
⇒ R= =
p 2.33 × 103
A = 6.25 × 10− 5 m

You might also like